Prelims Test 23 - GS Test 16-11-Mar-24 12 - 18 - 50

You might also like

Download as pdf or txt
Download as pdf or txt
You are on page 1of 76

Total Marks : 200.

00
Prelims Test 23 - GS Test 16
( Insta Prelims Test Series 2024 7.0 ) Mark Scored : 86.90

1. Consider the following pairs


Rebel Group Country which it belongs to
1. RED-Tabara Tajikistan
2. Houthi Yemen
3. Ta’ang National North Korea
Liberation Army

How many of the above pairs have been correctly matched?


A. Only one pair
B. Only two pairs
C. All three pairs
D. None

Your Answer :
7
Correct Answer : A
277
6 3
Answer Justification : 5 60
RED-Tabara is an armed rebel group of Burundi based in South - 9Kivu, Eastern
Congo, Africa.
.c om
ai l
Recently at least 20 people have been killed in Burundi following an attack by them.
The attacks targeted 9 homes in the Western town ;of m
gVugizo, close to the Lake
4
Tanganyika border with Congo. #6 &
5
Hence pair 1 is incorrect . va9
ta
iv
Houthis are Yemeni rebelrgroups.
as
ti s
The roots of the Houthiumovement can be traced to Believing Youth- a Zaydi- a revivalist
h r
Group founded by sHussain Al -Houthi and his father in early 1990s; they called
a - of God, mobilizing tribesman in the north against the government.
themselves Partitions
v
a Houthis seized an Israel linked ship bound for India which raised concerns of
Recently,stthe
a
riv dimension being added to the ongoing Gaza conflict.
another
S
i Ghaza-Israel conflict is a localised part of the Israeli -Palestinian conflict which
r ut The
Sh originated with withdrawal of Israel from Gaza in 2005.
Israel has claimed that the ship is British owned and Japanese operated.

Hence pair 2 is correct .

The Ta’ang National liberation army belongs to Myanmar.


Along with the Arakan Army and Myanmar National Democratic Alliance Army, it forms
the Three Brotherhood Alliance.
Recently the groups were in news as an entire Myanmar army battalion in the country’s
north west, surrendered to them.
The brotherhood Alliance expect to capture Laukkiang, the area’s major City.

Hence pair 3 is incorrect

instacourses.insightsonindia.com 1
© Insights Active Learning | All rights reserved - 18107. You may not reproduce, distribute or exploit the contents in any
form without written permission by copyright owner. Copyright infringers may face civil and criminal liability
Total Marks : 200.00
Prelims Test 23 - GS Test 16
( Insta Prelims Test Series 2024 7.0 ) Mark Scored : 86.90

2. The Indo-Pacific Regional Dialogue (IPRD) is organised by

A. India Trade Promotion Organisation


B. National Maritime Foundation
C. Institute for Defence Studies and Analyses
D. NITI Aayog

Your Answer : A
Correct Answer : B

Answer Justification :
The annual Indo-Pacific Regional Dialogue is organised by the National Maritime
Foundation, New Delhi as the Indian Navy’s knowledge partner.
the recent 2023 conference was a three day event in which Globally renowned experts 7
from India and abroad, senior officers from the Indian Armed Forces and the Government 2 77
3
06 sub
of India, scholars and public at large engaged in intensive deliberations in several
6
topics under the theme of ‘Geo political impacts upon Indo-Pacific Maritime
- 95 trade and
connectivity. m
othink
the Foundation signed MoUs of cooperation with four prominent
i l .c tanks in the Indo
m a
Pacific namely the Nepal Institute for international Cooperation and Engagement, Nepal;
the Global Centre for Policy and Strategy, Kenya ; 4 ;g
# 6 the Diplomatic Academy of Vietnam
and The Energy and Resources Institute, New &Delhi. 5
va9
Hence option B is correct. ta
i vas
i sr
r ut
sh
3. Consider the following-statements regarding International Tropical Timber
Organisation v a
a stathe International Tropical Timber Agreement, 2006.
1. It operates under
Sriv by the International Tropical Timber Council along with four committees that
2. It is governed
i
ut to develop forest related policies.
meets
r
ShIt aims for long time increase in consumption and continuity of supplies, in the international
3.
market, of tropical Timber from sustainable sources.

How many of the above statements are correct?


A. Only one
B. Only two
C. All three
D. None

Your Answer : C
Correct Answer : C

Answer Justification :
The International Tropical Timber Agreement, 2006 is an International Treaty
under which ITTO operates.

instacourses.insightsonindia.com 2
© Insights Active Learning | All rights reserved - 18107. You may not reproduce, distribute or exploit the contents in any
form without written permission by copyright owner. Copyright infringers may face civil and criminal liability
Total Marks : 200.00
Prelims Test 23 - GS Test 16
( Insta Prelims Test Series 2024 7.0 ) Mark Scored : 86.90

The Treaty entered into force on December 7, 2011, super ceding the International
Tropical Timber Agreement 1994.

Hence statement 1 is correct.

ITTO is governed by the International Tropical Timber Council which comprises


all the members of the organisation and meets once a year to develop forest related
policies and approve finance field level projects.
The council is assisted by four committees viz., the Committee on Economics,
Statistics and Markets, the Committee on Reforestation and Forest
Management, the Committee on Forest Industry and the Committee on Finance
and Administration.
The committees convene during sessions of the Council.

Hence statement 2 is correct.


7
2 77
63
The organisation provides for an effective Framework for consultation, International
0
cooperation and policy development among all members with regard to 5 6
all relevant
aspects of world Timber economy. -9
It strives to promote expansion and diversification of international
.c omtrade in tropical Timber
l
ai in the international markets
from sustainable sources by improving structural conditions
m
;g in consumption and continuity
by taking into account, on one hand, a long term increase
4
6the cost of Sustainable forest
&#equitable for members and the
of supplies and on the other, prices which reflect
management and which are remunerative 5 and
improvement of market access. va9
st a
v a
Hence statement 3 is correct.sri
i
r ut
- sh
4. Consider the following t a va statements regarding Indo- Pacific Economic Framework for
Prosperity (IPEF)i v as
1. It is t Sr trade agreement initiated by India along with countries of Indo-Pacific region.
aifree
ubased on four pillars including trade, supply chain resilience, infrastructure and tax.
2. hItris
S
3. It accounts to more than 10% of global trade in goods and services.

How many of the above statements are correct?


A. Only one
B. Only two
C. All three
D. None

Your Answer : B
Correct Answer : B

Answer Justification :
The Indo-Pacific Economic Framework was launched by US and other countries
of the Indo Pacific region in 2022.

instacourses.insightsonindia.com 3
© Insights Active Learning | All rights reserved - 18107. You may not reproduce, distribute or exploit the contents in any
form without written permission by copyright owner. Copyright infringers may face civil and criminal liability
Total Marks : 200.00
Prelims Test 23 - GS Test 16
( Insta Prelims Test Series 2024 7.0 ) Mark Scored : 86.90

It was first talked about in the East Asia summit of 2021 by US president Biden.
There are 14 members at present including Australia, India, Brunei , Fiji, Indonesia , Japan,
South Korea , Malaysia, New Zealand , Philippines, Singapore , Thailand, Vietnam and
USA.
It is not a traditional free trade agreement but the members are allowed to
negotiate the parts they decide to.

Hence statement 1 is incorrect .

It is not a trade agreement but includes different modules covering various aspects the four
pillars of the Framework are

Pillar I : Fair and resilient trade


Pillar II: supply chain resilience
Pillar III : infrastructure and decarbonization
7
Pillar IV: tax and anti corruption.
277
6 3
Hence statement 2 is correct 560
- 9
The Framework is seen as crucial as it accounts for 40% of the o m GDP and 20% of
world’s
l .c
the world’s trade in goods and services.
m ai
4 ;g
India has joined three pillars viz., the supply chain, decarbonization and infrastructure and
anti corruption and tax pillars. 6
&# in order to be a part of the trade pillar
Recently India has asked for better market5access
9
of the framework. va
a sta
Hence statement 3 is correct riv
u tis
s hr
v a-
5. With reference to
statements: va
sta‘Lantana Camara’ recently seen in news, consider the following
Sri one of the 10 worst invasive species in the world.
i
1. It is among
ut introduced in the country as an ornamental plant by the Portuguese in the 1800s.
2. Itrwas
Sh
3. Eradication of Lantana is much easier as compared to other invasive plant species.

How many of the above statements are correct?


A. Only one
B. Only two
C. All three
D. None

Your Answer :
Correct Answer : A

Answer Justification :

Only statement 1 is correct.

instacourses.insightsonindia.com 4
© Insights Active Learning | All rights reserved - 18107. You may not reproduce, distribute or exploit the contents in any
form without written permission by copyright owner. Copyright infringers may face civil and criminal liability
Total Marks : 200.00
Prelims Test 23 - GS Test 16
( Insta Prelims Test Series 2024 7.0 ) Mark Scored : 86.90

Lantana is not native to India. It was introduced in the country as an ornamental plant by
the British in the 1800s, and has since spread over 574,186 sq km, covering 50 per cent of the
country’s “natural areas”, according to an October 2023 study in the Journal of Applied Ecology.
A 2020 estimate by researchers, published in the journal Global Ecology and Conservation,
shows that the plant occupies 40 per cent of forests (over 154,800 sq km), including tiger
reserves.

Eradicating lantana is difficult “because of its rapid spread, intensity of infestation, allelopathy
[chemicals released to discourage growth of native plants], opportunistic growth behaviour,
reproductivity biology traits, and tenaious resistance to cutting and burning,” states the review
paper.

Source: https://www.downtoearth.org.in/news/forests/conquest-of-lantana-93453

7
277
6. Consider the following pairs 6 3
Reports Published by
560
1. International International Organisation for - 9
Migration Outlook Migration
.c om
2. Global Peace Index World Bank
ai l
3. World Drug Report UN Office on Drugs and m
Crime 4;g
& #6
How many of the above pairs are correct? a9
5
A. Only one pair s tav
B. Only two pairs r i va
C. All three pairs u tis
r
D. None sh
v a-
Bta
Your Answer : s
a
riv : A
Correct Answer
S
u ti
r
ShAnswer Justification :
The International Migration Outlook is published by the Organisation for
Economic Cooperation and Development.
The 2023 edition of the report analyses recent developments in migration movements
and the labour market inclusion of immigrants in OECD countries.
It also monitors recent policy changes in migration governance and integration in OECD
countries.

Hence pair 1 is incorrect.

The Global Peace Index is an annual publication of the Institute for Economics
and Peace.
The recently published 2023 edition ranked 163 independent States and territories
according to their level of peacefulness across the three domains of societal safety and
security; ongoing domestic and international conflict and Militarisation

instacourses.insightsonindia.com 5
© Insights Active Learning | All rights reserved - 18107. You may not reproduce, distribute or exploit the contents in any
form without written permission by copyright owner. Copyright infringers may face civil and criminal liability
Total Marks : 200.00
Prelims Test 23 - GS Test 16
( Insta Prelims Test Series 2024 7.0 ) Mark Scored : 86.90

it also states that the average level of peacefulness has decreased by point 0.42%
globally
Iceland is the most peaceful country in the world while Afghanistan is the least peaceful
India holds 126th rank.

Hence pair 2 is incorrect.

The World Drug Report is published by the United Nations Office on Drugs and
Crime.
It is an annual publication that highlights the expanding elicit drug Markets and the
challenges they pose to health services and law enforcement.

Hence pair 3 is correct

7
277
7. Consider the following statements regarding Global Biofuels Alliance 3
1. It has been launched by India at the G20 summit. 6 06
5
-9
2. It will collaborate with the World Economic Forum to promote sustainable biofuels.
om
3. It Is important for achievement of UN Sustainable Development Goal-7.
ail.c
How many of the above statements are correct? ;g m
4
A. Only one
& #6
B. Only two 5
va9
C. All three
ta
D. None as iv
sr
u ti
Your Answer : B
shr
Correct Answer : C -
t a va
i v as
Answer Justification :
r
S Global Biofuels Alliance is an India led initiative that intends to develop an Alliance of
t iThe
ru national governments, industries and international organisations to promote the adoption
Sh of biofuels.
It was launched at the 18th G20 summit held in New Delhi in September 2023
it was the first time India hosted the G20 leaders summit with the ‘ the world is one
family’.
The alliance was launched with nine initiating members – India, US, Brazil, Argentina,
Bangladesh, Italy , Mauritius, South Africa and the UAE.

Hence statement 1 is correct

The initiative brings together 19 countries and 12 International organisations including


the World Economic Forum , Asian Development Bank, World LPG organisation, Biofutures
Platform International Energy Agency , World Biogas Association the World Bank and
others to try to expand the use of sustainable biofuels.
The World Economic Forum is a member of the alliance, and its report on
Fostering Effective Energy Transition highlights how countries such as Brazil

instacourses.insightsonindia.com 6
© Insights Active Learning | All rights reserved - 18107. You may not reproduce, distribute or exploit the contents in any
form without written permission by copyright owner. Copyright infringers may face civil and criminal liability
Total Marks : 200.00
Prelims Test 23 - GS Test 16
( Insta Prelims Test Series 2024 7.0 ) Mark Scored : 86.90

are using policies to expand biofuel use.

Hence statement 2 is correct.

The alliance will be instrumental to the achievement of several UN Sustainable


Development Goals in particular SDG-7 ( provide access to affordable, reliable,
sustainable and modern energy for everyone).
the objectives of the Alliance also supports the decarbonization efforts taken by countries
to meet climate action imperatives.

Hence statement 3 is correct.

8. Consider the following statements regarding India- UN Capacity Building Initiative


7
77of
1. Under it, the UN-India Team and the Gates Foundation will collaborate to enhance capacity
2
countries in the Global South. 3
6
2. It aims to promote the G20 Action plan for Advancing the Sustainable Development06Goals and
5
digital infrastructure. -9
.c om
Which of the above statements is/are correct? ail
A. 1 only
4 ;gm
B. 2 only
& #6
C. Both 1 and 2 5
D. Neither 1 nor 2 va9
a st
va
Your Answer : B i sri
u t
Correct Answer : C
shr
-
Answer Justification
t a va :
i v as the United Nations have jointly launched an initiative, the India-UN Capacity
India and

i Sr Initiative that intends to build capacity of countries in the Global South


Building
t
h ru the goal is to share India’s development experiences, best practices and expertise
S through capacity building and training programs.
The initiative builds upon extensive cooperation in development and capacity building
that India already has bilaterally, with partner countries.
Under this capacity building programme, the UN India team and the Millinda Gates
Foundation will collaborate utilising India’s technical and economic cooperation platform
to share India’s development experiences and best practices.

Hence statement 1 is correct.

The initiative extends the success of India’s G20 presidency which prioritised the inclusion
of voices from the Global South in the G20 agenda.
The initiative will put into action, the development goals established during India's G20
presidency, including the G20 Action Plan for advancing the Sustainable Development
Goals and enhancing technology and digital infrastructure.

instacourses.insightsonindia.com 7
© Insights Active Learning | All rights reserved - 18107. You may not reproduce, distribute or exploit the contents in any
form without written permission by copyright owner. Copyright infringers may face civil and criminal liability
Total Marks : 200.00
Prelims Test 23 - GS Test 16
( Insta Prelims Test Series 2024 7.0 ) Mark Scored : 86.90

Hence statement 2 is correct

9. Washington Declaration, recently in news, is related to

A. Monitoring of natural disasters and related response management


B. Nuclear deterrence strategy
C. Promotion of electric vehicles
D. Guidelines for immigrants

Your Answer : D
Correct Answer : B

Answer Justification : 7
South Korean president Yoon Suk Yeol visited US to celebrate 70th anniversary of2US- 77
3
South Korea bilateral relations recently
6 06
during the visit , the two countries signed the Washington declaration
- 95 which
focuses on nuclear deterrence strategy.
.c om a joint nuclear
l
The agreement has been signed between the two nations outlining
ai two countries will take in
deterrence strategy and specifies several measures that
;g m the
cooporation towards deterrence including : 4
1. deployment of an American ballistic submarine & #in6the Korean Peninsula
5
2. formation of a nuclear consultative group
va9to formulate principles of joint response tactics
sta deterrence capabilities.
3. strengthening of South Korean nuclear
a
The declaration reaffirms that
sr ivSouth Korea would not make its own nuclear capabilities
i deterrence measures through an alliance based approach.
and would instead focus ton
u
shr
Hence option B is a -
correct.
v
a sta
Sriv
i
ut the following statements regarding United Nations Human Rights Council
10. Consider
r
ShIt was established by the UN General Assembly replacing the UN Commission on Human Rights.
1.
2. The Office of High Commissioner for Human Rights acts as the secretariat for the Council.
3. Two thirds of the seats in the Council are reserved for Latin America and European region and
the rest are equally divided among the other UN regional groups.
4. It organises the Nelson Mandela Moot Court Competition globally.

How many of the above statements are correct?


A. Only one
B. Only two
C. Only three
D. All four

Your Answer : C
Correct Answer : C

instacourses.insightsonindia.com 8
© Insights Active Learning | All rights reserved - 18107. You may not reproduce, distribute or exploit the contents in any
form without written permission by copyright owner. Copyright infringers may face civil and criminal liability
Total Marks : 200.00
Prelims Test 23 - GS Test 16
( Insta Prelims Test Series 2024 7.0 ) Mark Scored : 86.90

Answer Justification :
The Human Rights Council is the main intergovernmental body within the United Nations
responsible for promotion of Human Rights globally
it was established by the General Assembly in 2006
It replaced the former United Nations Commission on Human Rights.
The Council is composed of 47 member states and provides a multilateral forum to
address human rights violations and country situations
it responds to human rights emergencies and makes recommendations on how to better
implement Human Rights on the ground
The Council benefits from substantive, technical and Secretariat support from the Office
of the High Commissioner for human rights.
The Council has the ability to discuss all thematic human rights issues and situations that
require its attention throughout the year
it meets at the United Nations Office at Geneva (UNOG).
7
Hence statements 1 and 2 are correct. 277
3
6 06
The United Nations organises its members States into five regions - Asia 5, Africa, Latin
America and the Caribbean, Eastern Europe, Western Europe and others. -9
The council consists of 47 member states elected directly and .c om
individually by a majority of
ai l
193 states of the UN General Assembly
Elections take place every year 4 ;gm
Seats are equitably distributed among the five#
6
UN regional groups with one third of the
&
5 membership is limited to consecutive terms.
members being renewed each year term9and
va
ta
Hence statement 3 is incorrect . vas
i sri
The Nelson Mandela r ut court competition is coorganized by the UN Human Rights
Moot
Council branch - s
h
a court competition from students all over the world that is explicitly
vMoot
a
it is the only
t
dedicated
i v as to human rights.
i Sr
t statement 4 is correct.
ru
Hence
Sh

11. Consider the following Nations


1. Italy
2. Turkey
3. Germany
4. France
5. Egypt
6. Saudi Arabia

Which of the above nations are members of India- Middle East- Europe Economic Corridor?
A. 2, 3, 5 and 7 only
B. 1, 3, 4 and 6 only
C. 2, 3, 4, 6 and 7 only

instacourses.insightsonindia.com 9
© Insights Active Learning | All rights reserved - 18107. You may not reproduce, distribute or exploit the contents in any
form without written permission by copyright owner. Copyright infringers may face civil and criminal liability
Total Marks : 200.00
Prelims Test 23 - GS Test 16
( Insta Prelims Test Series 2024 7.0 ) Mark Scored : 86.90

D. 1, 2, 5 and 6 only

Your Answer : D
Correct Answer : B

Answer Justification :
The India-Middle East -Europe Economic corridor includes India, the UAE, Saudi
Arabia, European Union, France, Italy, Germany and the US.
The project has been launched by the Prime Minister of India with an aim to enable
greater trade among the partner Nations including energy products.
The project consists of an Eastern Corridor connecting India to the Gulf region and a
Northern corridor connecting Gulf region to Europe
it also includes are railway and ship- rail transit Network and Road Transport routes
the corridor will include a Rail link as well as an electricity cable, a hydrogen pipeline and
a high speed data cable.
7 77
The rail and shipping corridor is part of the Partnership for Global Infrastructure3 2
Investment. 6 06
5
- 9 an alternative
It is seen as a response to the Belt and Road initiative of China, providing
infrastructure network. om
ail.c
Hence option B is correct.
;g m
6 4
5 &#
va9
12. Consider the following pairs
ta
Reports Published by
i vas
1. Global Gender
i r
World sBank
t
Gap Report ru
2. Global Liveability sh Human Rights Watch
Index v a-
a sta
3. Global Competitiveness World Trade Organisation
Index r i v
i S
r ut
ShHow many of the above pairs are correct?
A. Only one pair
B. Only two pairs
C. All three pairs
D. None

Your Answer : A
Correct Answer : D

Answer Justification :
The Global Gender Gap report is released by the World Economic Forum.
It is an annual publication that benchmarks the current state and evolution of gender
parity across four key dimensions viz., Economic participation and opportunity;
Educational attainment; Health and survival and Political empowerment.
It tracks the progress of numerous countries efforts towards closing these gaps overtime

instacourses.insightsonindia.com 10
© Insights Active Learning | All rights reserved - 18107. You may not reproduce, distribute or exploit the contents in any
form without written permission by copyright owner. Copyright infringers may face civil and criminal liability
Total Marks : 200.00
Prelims Test 23 - GS Test 16
( Insta Prelims Test Series 2024 7.0 ) Mark Scored : 86.90

since its inception in 2006.


In the report published in 2023, India has been ranked at 127 out of 146 countries in
terms of gender parity.

Hence pair 1 is correct.

The Global Liveability index is published by The Economist Intelligence Unit.


The index rans living conditions in 173 cities globally using five categories which include
healthcare, culture and environment, stability, education and infrastructure
in the recently published index (2023), Vienna, Austria has been placed at the top
position.

Hence pair 2 is incorrect.

The Global Competitiveness Index is released by the International Institute for


7
Management Development, an independent University institute based in Switzerland
277
and Singapore.
0 63
th
In the recent edition (2023), India has been ranked 40 - a drop of three 56
spots from the
previous year -9
om
The top three positions are occupied by Denmark, Ireland andcSwitzerland.
ail.
Hence pair 3 is incorrect ;g m
6 4
5 &#
va9
13. Consider the following statements regarding
asta International Sugar Organisation
ri
1. It promotes use of Biofuels by emphasizingv on including ethanol from sugar crops in its
ti s
rufor 100% of world export and import of sugar.
activities.
h
-s
2. Its members are responsible
v a
3. It administers the International Sugar Agreement 1992 with a focus on non-traditional usage of
t a
sugar.
i v as worldwide fuel and non-fuel statistics of ethanol annually.
Sr
4. It publishes the
t i
h ru many of the above statements are correct?
How
S A. Only one
B. Only two
C. Only three
D. All four

Your Answer : C
Correct Answer : C

Answer Justification :
The International Sugar Organisation is the unique intergovernmental body devoted to
improving conditions on the world's sugar market through analysis, debate, studies ,
transparent statistics , conferences and workshops.
The proactive attitude to include ethanol from sugar crops in its activities in the mid 90’s
has helped to promote the increasing role of Biofuels in the future energy-mix worldwide.

instacourses.insightsonindia.com 11
© Insights Active Learning | All rights reserved - 18107. You may not reproduce, distribute or exploit the contents in any
form without written permission by copyright owner. Copyright infringers may face civil and criminal liability
Total Marks : 200.00
Prelims Test 23 - GS Test 16
( Insta Prelims Test Series 2024 7.0 ) Mark Scored : 86.90

It is based in London.

Hence statement 1 is correct.

The ISO consists of 87 member states (2022) which represent :

87% of world sugar production


64% of world sugar consumption
34% of world imports
92% of world exports

Recently Saudi Arabia has become the 88th member of ISO.

India will chair ISO in 2024.

7
Hence statement 2 is incorrect.
277
3
06 Sugar
The ISO exists to administer the internationally negotiated 1992 International
6
Agreement the objectives of which are, to : 95 -
m
.co and related issues
Ensure enhanced International cooperation regarding sugarlmatters
i
Provide a forum for intergovernmental consultations onm a and ways to improve world
Sugar
sugar economy 4;g
6
5 &#
Facilitate trade by collecting and providing information on the world sugar market and
other sweeteners
va9
a
st particularly non traditional uses.
Encourage increased demand for sugar
va
Hence statement 3 is correct. i sri
t
h ru
The ISO publishes
a - s the Ethanol Yearbook, which provides comprehensive statistics of
tav ( fuel and non fuel) market including nearly 100 tables comprising
the world ethanol
s
iva consumption, trade, prices as well as government policy.
production,
r
t i Sstatement 4 is correct.
ru
Hence
Sh

14. Consider the following statements regarding UN Relief and Works Agency for
Palestine refugees in the near east (UNRWA)
1. It provides humanitarian relief particularly to Palestinians who became refugees during the
1948 Palestine war.
2. Its operational area includes all the countries in the Middle East and Africa.
3. It is completely funded by United Nations General Assembly budget funds.

How many of the above statements are correct?


A. Only one
B. Only two
C. All three
D. None

instacourses.insightsonindia.com 12
© Insights Active Learning | All rights reserved - 18107. You may not reproduce, distribute or exploit the contents in any
form without written permission by copyright owner. Copyright infringers may face civil and criminal liability
Total Marks : 200.00
Prelims Test 23 - GS Test 16
( Insta Prelims Test Series 2024 7.0 ) Mark Scored : 86.90

Your Answer : A
Correct Answer : A

Answer Justification :
UNRWA is the sole United Nations agency that supports relief and human development of
Palestinian refugees
it was established in 1949 when nearly three quarters of a million Palestinians became
refugees after the 1948 Arab Israeli war.

Hence statement 1 is correct.

It was originally headquartered in Lebanon but in 1978 was shifted to Vienna, Austria
in 1996 the UN General Assembly move the agency to Gaza strip to demonstrate it's
commitment to the Arab Israeli peace process.
At present it provides services in five fields of operations- Lebanon, Jordan, 7
Syria , Gaza strip, and West Bank including east of Jerusalem. 2 77
3
It has offices in New York, Washington , Brussels and Cairo. 06
9 56
Hence statement 2 is incorrect. -
.c om
ai l
The organisation gets its funding from voluntary contributions by UN member
states including regional governments and the;g m
European Union
these sources represent about 93% of financial# 64 contributions to the agency
&
95 local tech companies, large
it also partners with businesses and foundations
a
tav
multinationals, NGOs, etc.
s
Hence statement 3 is incorrect r i va
u tis
shr
v a-
a sta
15. Consider the following statements regarding International Competition Network
1. It was setrupiv on recommendations of International Competition Policy Advisory Committee.
S
ti competition authorities reach consensus on its recommendations, instead of involving
2. It helps
u
r
Shitself into rule making function.
3. The Competition Commission of India was set up under its mandate.

How many of the above statements are correct?


A. Only one
B. Only two
C. All three
D. None

Your Answer : B
Correct Answer : B

Answer Justification :
The concept for the International Competition Network originated out of the
recommendations made by the International Competition Policy Advisory Committee 1997

instacourses.insightsonindia.com 13
© Insights Active Learning | All rights reserved - 18107. You may not reproduce, distribute or exploit the contents in any
form without written permission by copyright owner. Copyright infringers may face civil and criminal liability
Total Marks : 200.00
Prelims Test 23 - GS Test 16
( Insta Prelims Test Series 2024 7.0 ) Mark Scored : 86.90

the committee was commissioned to address Global anti -trust problems in the context of
economic globalization and focused on issues including multi jurisdictional merger review,
interface between trade and competition and the future direction for cooperation between
anti trust agencies.
Finally after certain negotiations in 2001 , top anti trust officials from 14 jurisdictions
launched the ICN at a meeting in the New York city.

Hence statement 1 is correct.

It is the only Global body devoted exclusively to competition law enforcement and its
members represent National and multinational competition authorities.
It does not exercise any rule making function
where it reaches consensus on recommendations arising from the projects, individual
competition authorities decide on whether and how to implement the recommendations
through unilateral , bilateral or multilateral agreements , as appropriate. 7
277
Hence statement 2 is correct. 6 3
560
The Competition Commission of India is a statutory body of the 9
- Government of
m
India established in 2009 under the competition Act 2002.
i l .co
a that will provide a level
It aims to create and sustain fair competition in the economy
m
playing field to producers and make markets work for
4 ;gwelfare of the consumers.
Recently the Competition Commission of India
& #6 has become a part of the 18
5
member Stearing committee of the International Competition Network.
va9
Hence statement 3 is incorrect ast
a
sr iv
i
r ut
- sh
16. Consider the following
v a statements
staNational Investment and Infrastructure Fund (NIIF) are shared equally by the
1. The stakes of the
a
riv and foreign investors.
Government
S
i Japan Fund is the first bilateral fund of NIIF.
2. The tIndia-
r u
3. hThe India-Japan Fund will finance low carbon emissions strategies.
S
How many of the above statements are correct?
A. Only one
B. Only two
C. All three
D. None

Your Answer : B
Correct Answer : B

Answer Justification :
The National Investment and Infrastructure Fund was formed in 2015
it is a government backed category II alternate investment fund created to offer long term
financial support to infrastructure sector in the country

instacourses.insightsonindia.com 14
© Insights Active Learning | All rights reserved - 18107. You may not reproduce, distribute or exploit the contents in any
form without written permission by copyright owner. Copyright infringers may face civil and criminal liability
Total Marks : 200.00
Prelims Test 23 - GS Test 16
( Insta Prelims Test Series 2024 7.0 ) Mark Scored : 86.90

the government holds 49% stake in the fund while the remaining shares are
owned by foreign and domestic investors.

Hence statement 1 is incorrect.

NIIF has partnered with the Japan Bank for International Cooperation to establish a $600
million India-Japan Fund with the Government of India and JBIC as primary investors.
It is NIIF’s first bilateral Fund with the Government of India contributing 49% of
the total fund while the remaining 51% has been contributed by JBIC.

Hence statement 2 is correct.

This collaboration reflects a significant joint effort between India and Japan in the climate
and environmental sustainability sector
The primary focus of the fund is on investments in Environmental sustainability
7
and low carbon emission strategies
277
63
it's objective is to become a preferred partner for encouraging increased Japanese
0
investment in India. 5 6
-9
Hence statement 3 is correct.
.c om
ail
4 ;gm
17. Which of the following Nations is not a member & #of6Abraham Accords, sometimes seen
5
in news?
va9
asta
A. Israel
sr iv
i
B. Bahrain
r ut
C. Morocco
- sh
D. Algeria
t a va
i v a: sC
r
Your Answer
SAnswer
i
r ut
Correct :D
ShAnswer Justification :
The Abraham records refer to the series of agreements that came into effect in 2020
between Israel and several Arabian States marking a historic shift in diplomatic relations
in the Middle East
the Accords have been named Abraham Accords as part the supposed common ancestor
of the Arabs and the Jews -the biblical Abraham and as an expression of brotherhood
among the Nations
the major countries involved in Abraham Accords that was mediated by United
States include Israel, UAE, Bahrain, Sudan, Morocco.
The agreement provides how the Arab countries are gradually shifting themselves from
the Palestine question
under it, full diplomatic relations will be established between Israel, UAE and Bahrain
which will have positive impact on the entire Middle East region.

instacourses.insightsonindia.com 15
© Insights Active Learning | All rights reserved - 18107. You may not reproduce, distribute or exploit the contents in any
form without written permission by copyright owner. Copyright infringers may face civil and criminal liability
Total Marks : 200.00
Prelims Test 23 - GS Test 16
( Insta Prelims Test Series 2024 7.0 ) Mark Scored : 86.90

Hence option D is correct.

18. Consider the following statements regarding World Health Assembly


1. It is the decision making body of World Health Organisation that focuses on a specific health
agenda.
2. It is responsible for supervising financial policies and provide final approval for Programme
budget.
3. The first ever Global Plan of Action for the Health of Indigenous People has been released at the
75th session of the Assembly.

How many of the above statements are correct?


A. Only one
B. Only two 7
C. All three 277
6 3
D. None
560
- 9
Your Answer : B m
Correct Answer : B
il.co
m a
Answer Justification : 4; g
6
The World Health Assembly is the decision
5 &#making body of World Health
Organisation
v a9
It is attended by delegations from a
st all member states of WHO
va
It focuses on a specific healthiagenda prepared by the Executive Board.
tisr
u
hr
Hence statement 1 is correct.
a -s
tav of the Assembly are to
The major functions
s
a
S riv policies of the organisation
determine
i
r ut appoint the director General
Sh supervise financial policies and
review and approve the proposed programme budget.

Hence statement 2 is correct.

That 76th annual World Health Assembly was organised at the World Health
Organisation’s headquarters at Geneva, Switzerland in May 2023
in this annual session a draft resolution has been accepted for developing a
Global plan of action for the Health of Indigenous people
the plan will be considered at the 79th World Health assembly in 2026.

Hence statement 3 is incorrect

instacourses.insightsonindia.com 16
© Insights Active Learning | All rights reserved - 18107. You may not reproduce, distribute or exploit the contents in any
form without written permission by copyright owner. Copyright infringers may face civil and criminal liability
Total Marks : 200.00
Prelims Test 23 - GS Test 16
( Insta Prelims Test Series 2024 7.0 ) Mark Scored : 86.90

19. Consider the following statements regarding the Vienna Convention on Diplomatic
Relations 1961
1. Under it, diplomats are provided protection against arrest and detention in a host country.
2. It also provides exemption to the diplomat from customs duties.
3. India is a member of this United Nations treaty.

How many of the above statements are correct?


A. Only one
B. Only two
C. All three
D. None

Your Answer : B
Correct Answer : C
7
277
Answer Justification : 3
The Vienna Convention on Diplomatic Relations 1961 defines fundamental 6 06
5
principles and terms governing how countries should treat diplomatic -9
representatives.
.c om
It was adopted in April 1961 by the United Nations a i l
Conference on Diplomatic
Intercourse and Immunities at Vienna, Austria.;g m
The Convention provides diplomatic immunity and
# 64grants exemption to diplomats from
certain laws and taxes in the country where
9 5&they are posted
a
tav
according to article 29 of the Convention diplomats are not subject to arrest or
detention a s
r i vprovide
the host country is required
ti s to all necessary measures to prevent any kind of
ruliberty or dignity of the diplomat.
harm or violation to the
hthe
-
Articles 34 and 36 s
of Convention provides that diplomats are exempted from
a
direct taxesv and custom duties that are levied by the country in which they are
a
posted.ast
Sriv
i
ut Statements 1 and 2 are correct.
Hence
r
Sh Till now 193 countries have ratified the convention
India ratified it through the Diplomatic Relations (Vienna Convention) act of
1972.
It was in news as recently In response to the allegations raised by the Canadian Foreign
Minister regarding India’s violation of diplomatic protocols, the Indian Ministry of External
Affairs provided that the country’s actions are consistent with the provisions outlined in
article 11.1 of the Vienna Convention on Diplomatic Relations.

Hence statement 3 is correct

20. Valmiki Tiger Reserve is located in which of the following state?

A. Uttarakhand
instacourses.insightsonindia.com 17
© Insights Active Learning | All rights reserved - 18107. You may not reproduce, distribute or exploit the contents in any
form without written permission by copyright owner. Copyright infringers may face civil and criminal liability
Total Marks : 200.00
Prelims Test 23 - GS Test 16
( Insta Prelims Test Series 2024 7.0 ) Mark Scored : 86.90

B. Uttar Pradesh
C. Bihar
D. Madhya Pradesh

Your Answer :
Correct Answer : C

Answer Justification :

Authorities in Bihar’s Valmiki Tiger Reserve (VTR) have credited the rise in the protected area
(PA)’s big cat numbers to a reduction in human activities, including restrictions on illegal mining
in its eco-sensitive zone.

VTR Field Director Neshmani K told this reporter that a total ban on sand and stone mining
7
inside VTR, and strict restrictions on mining in its eco-sensitive zone, helped increase grassland
277
cover. This, in turn, caused tiger numbers in the PA to rise to 54 earlier this year, up from 31 in
3
2018. In 2014, there were only 28 tigers in VTR. 06
9 56
Source: -
.c om
https://www.downtoearth.org.in/news/mining/bihar-reduced-human-activity-including-mining-ba
ns-helped-increase-tiger-numbers-in-valmiki-reserve-93485 ai l
;g m
6 4
&#
95
ta va
21. With reference to the Chief Election Commissioner and Other Election Commissioners
(Appointment, Conditions of Servicesand Term of Office) Act, 2023, consider the
following statements: r i va
u tis by the President upon the recommendation of a Selection
1. The CEC and ECs will be appointed
Committee. shr
v a - will consist of the Prime Minister, CJI and the Leader of
2. The Selection Committee
staof the largest opposition party in Lok Sabha.
Opposition/leader
a
iv
3. In case ofrvacancy in the committee, recommendations of the Selection Committee will not be
i S
ut
valid.
r
Sh
How many of the above statements are correct?
A. Only one
B. Only two
C. All three
D. None

Your Answer : A
Correct Answer : A

Answer Justification :

Appointment Process under Chief Election Commissioner and Other Election Commissioners
(Appointment, Conditions of Service and Term of Office) Act, 2023:

instacourses.insightsonindia.com 18
© Insights Active Learning | All rights reserved - 18107. You may not reproduce, distribute or exploit the contents in any
form without written permission by copyright owner. Copyright infringers may face civil and criminal liability
Total Marks : 200.00
Prelims Test 23 - GS Test 16
( Insta Prelims Test Series 2024 7.0 ) Mark Scored : 86.90

The CEC and ECs will be appointed by the President upon the recommendation of a
Selection Committee.

Hence, statement 1 is correct.

The Selection Committee will consist of the Prime Minister, a Union Cabinet Minister, and
the Leader of Opposition/leader of the largest opposition party in Lok Sabha.

7
Hence, statement 2 is incorrect. 277
6 3
560
- 9
m
Recommendations of the Selection Committee will be valid even
i l .co when there is a vacancy
in this Committee.
m a
4; g
6
5 &#
Hence, statement 3 is incorrect. va9
sta
va
i sri
t
h ruPublic Service Commission (SPSC), consider the following
-s
22. With reference to the State
statements: a
1. On ceasing to s tavoffice, the chairman of a SPSC is not eligible for appointment as the
hold
chairmanror ivaa member of the UPSC.
i S
ua tmember of the UPSC.
2. On ceasing to hold office, a member of a SPSC is not eligible for appointment as the chairman
r
Sh
or

Which of the above statement is/are correct?


A. 1 only
B. 2 only
C. Both 1 and 2
D. Neither 1 nor 2

Your Answer : D
Correct Answer : D

Answer Justification :

The chairman of a SPSC (on ceasing to hold office) is eligible for appointment as the
chairman or a member of the UPSC or as the chairman of any other SPSC, but not for any other

instacourses.insightsonindia.com 19
© Insights Active Learning | All rights reserved - 18107. You may not reproduce, distribute or exploit the contents in any
form without written permission by copyright owner. Copyright infringers may face civil and criminal liability
Total Marks : 200.00
Prelims Test 23 - GS Test 16
( Insta Prelims Test Series 2024 7.0 ) Mark Scored : 86.90

employment under the Government of India or a state.

Hence, statement 1 is incorrect.

A member of a SPSC (on ceasing to hold office) is eligible for appointment as the chairman
or a member of the UPSC, or as the chairman of that SPSC or any other SPSC, but not for any
other employment under the Government of India or a state.

Hence, statement 2 is incorrect.

23. Which one of the followings is not a function of Finance Commission:

A. To make the recommendations regarding the distribution of the net proceeds of taxes to
7
be shared between the Centre and the states.
2 77
B. To make the recommendations regarding the principles that should govern the 3grants-in-
aid to the states by the Centre. 6 06
5
-9
C. To make the recommendations regarding the measures needed to augment the
m
consolidated fund of a state to supplement the resources of theopanchayats and the
municipalities in the state a il.c
D. None of these ;gm
#64
Your Answer : D 5 &
Correct Answer : D va9
ta
i vas
Answer Justification :
i sr
r ut
The Finance Commissionsis h required to make recommendations to the president of India on the
-
following matters: va
a sta
Sriv
1. The distribution of the net proceeds of taxes to be shared between the Centre and the
i
ut and the allocation between the states of the respective shares of such proceeds.
states,
r
ShHence, option (a) is correct.

2. The principles that should govern the grants-in-aid to the states by the Centre (i.e., out
of the consolidated fund of India).

Hence, option (b) is correct.

3. The measures needed to augment the consolidated fund of a state to supplement the
resources of the panchayats and the municipalities in the state on the basis of the
recommendations made by the state finance commission.

Hence, option (c) is correct.

instacourses.insightsonindia.com 20
© Insights Active Learning | All rights reserved - 18107. You may not reproduce, distribute or exploit the contents in any
form without written permission by copyright owner. Copyright infringers may face civil and criminal liability
Total Marks : 200.00
Prelims Test 23 - GS Test 16
( Insta Prelims Test Series 2024 7.0 ) Mark Scored : 86.90

4. Any other matter referred to it by the president in the interests of sound finance.

Hence, option (d) is the correct answer as all the statements are the functions of
Finance Commission.

24. Consider the following statements regarding the working of the GST Council:
1. Two-third of the total number of members of the Council is the quorum for conducting a
meeting.
2. Every decision of the Council is to be taken by a majority of not less than three-fourths of the
weighted votes of the members present and voting at the meeting.
3. The vote of the central government shall have a weightage of two-third of the total votes cast in
that meeting.
7
How many of the above statements are incorrect? 277
A. Only one 6 3
B. Only two 560
- 9
C. All three m
D. None
il.co
m a
Your Answer : C 4 ;g
6
Correct Answer : B
5 &#
va9
Answer Justification : ta
i vas
WORKING OF THE GST COUNCIL
tisr
h ru
- s
The decisions of the Council are taken at its meetings. One-half of the total number of
v a
members of theaCouncil is the quorum for conducting a meeting.
a st
riv
Hence,Sstatement 1 is incorrect.
t i
h ru decision of the Council is to be taken by a majority of not less than three-fourths of the
Every
S
weighted votes of the members present and voting at the meeting.

Hence, statement 2 is correct.

The decision is taken in accordance with the following principles:

(i) The vote of the central government shall have a weightage one-third of the total votes cast
in that meeting.

instacourses.insightsonindia.com 21
© Insights Active Learning | All rights reserved - 18107. You may not reproduce, distribute or exploit the contents in any
form without written permission by copyright owner. Copyright infringers may face civil and criminal liability
Total Marks : 200.00
Prelims Test 23 - GS Test 16
( Insta Prelims Test Series 2024 7.0 ) Mark Scored : 86.90

Hence, statement 3 is incorrect.

(ii) The votes of all the state governments combined shall have weightage of two-thirds of the
total votes cast in that meeting.

Any act or proceedings of the Council will not become invalid on the following grounds:

(i) any vacancy or defect in the constitution of the Council; or

(ii) any defect in the appointment of a person as a member of the 7


277
6 3
60
Council; or
9 5
(iii) any procedural irregularity of the Council not affecting the merits -
m
il.co
of the case.
m a
4 ;g
6
5 &#
a9
25. With reference to the Special Officer forvLinguistic Minorities, consider the following
ta
as (1953–55) which made recommendations regarding
statements:
r i
1. It was States Reorganization Commissionv
the establishment of Special u tis for Linguistic Minorities.
Officer
r India did not make any provision with respect to the Special
hof
s
2. Originally, the Constitution
-
vaMinorities.
Officer for Linguistic
a sta
iv
Which of rthe above statement is/are incorrect?
S
A.ti1 only
ru
Sh B. 2 only
C. Both 1 and 2
D. Neither 1 nor 2

Your Answer : C
Correct Answer : D

Answer Justification :

Originally, the Constitution of India did not make any provision with respect to the Special
Officer for Linguistic Minorities.

Hence, statement 2 is correct.

Later, the States Reorganisation Commission (1953–55) made a recommendation in this

instacourses.insightsonindia.com 22
© Insights Active Learning | All rights reserved - 18107. You may not reproduce, distribute or exploit the contents in any
form without written permission by copyright owner. Copyright infringers may face civil and criminal liability
Total Marks : 200.00
Prelims Test 23 - GS Test 16
( Insta Prelims Test Series 2024 7.0 ) Mark Scored : 86.90

regard. Accordingly, the Seventh Constitutional Amendment Act of 1956 inserted a new Article
350- B in Part XVII of the Consti-tution.

Hence, statement 1 is correct.

26. Consider the following statements regarding the Chief Information Commissioner:
1. The Chief Information Commissioner and information Commissioner shall hold office during the
pleasure of President of India.
2. The salary, allowances and other service conditions of the Chief Information Commissioner and
an Information Commissioner shall be such as prescribed by the Parliament of India.
3. The Commission consists of a Chief Information Commissioner and not more than ten
Information Commissioners.
7
How many of the above statements are incorrect? 277
A. Only one 6 3
B. Only two 560
- 9
C. All three
m
D. None
il.co
m a
Your Answer : B 4 ;g
6
Correct Answer : B
5 &#
va9
Answer Justification : ta
i vas
isr and an Information Commissioner shall hold office for
The Chief Information Commissioner
t
such term as prescribed rby u
sh the Central Government or until they attain the age of 65
a-
years, whichever is earlier.
v
a
Hence, statementsta 1 is incorrect.
S riv
i
The
r utsalary, allowances and other service conditions of the Chief Information Commissioner and
Shan Information Commissioner shall be such as prescribed by the Central Government. But,
they cannot be varied to his disadvantage during service.

Hence, statement 2 is incorrect.

The Commission consists of a Chief Information Commissioner and not more than ten
Information Commissioners.

Hence, statement 3 is correct.

27. With reference to the Central Bureau of Investigation (CBI), Consider the following
statements:
1. The Central Bureau of Investigation (CBI) was set up in 1963 by a legislation passed by the
Parliament of India.

instacourses.insightsonindia.com 23
© Insights Active Learning | All rights reserved - 18107. You may not reproduce, distribute or exploit the contents in any
form without written permission by copyright owner. Copyright infringers may face civil and criminal liability
Total Marks : 200.00
Prelims Test 23 - GS Test 16
( Insta Prelims Test Series 2024 7.0 ) Mark Scored : 86.90

2. The establishment of the CBI was recommended by the Santhanam Committee on Prevention of
Corruption.
3. The CBI is a statutory body.

How many of the above statements are correct?


A. Only one
B. Only two
C. All three
D. None

Your Answer : C
Correct Answer : A

Answer Justification :
7
ESTABLISHMENT OF CBI 277
3
6 06
The Central Bureau of Investigation (CBI) was set up in 1963 by a resolution
- 95 of the Ministry
om
of Home Affairs.
l .c
Hence, statement 1 is incorrect.
m ai
4 ;g
Later, it was transferred to the Ministry of Personnel #6 now it enjoys the status of an attached
and
&
5 into vigilance cases) setup in 1941 was
office. The Special Police Establishment (which9looked
va
sta
also merged with the CBI.
a
iv recommended by the Santhanam Committee on
The establishment of the CBI swas
i r
ut
Prevention of Corruptionr(1962–1964). The CBI is not a statutory body. It derives its
sh
powers from the Delhi -Special Police Establishment Act, 1946.

t a va
i v as
Hence, statement 2 is correct but statement 3 is incorrect.

i Sr
t
h ru
S
28. Consider the following statements:
1. The National Investigation Agency (NIA) was constituted in 2009 under the provisions of the
National Investigation Agency Act, 2008.
2. It is the central counter-terrorism law enforcement agency in the country.

Which of the above statement is/are correct?


A. 1 only
B. 2 only
C. Both 1 and 2
D. Neither 1 nor 2

Your Answer : C
Correct Answer : C

instacourses.insightsonindia.com 24
© Insights Active Learning | All rights reserved - 18107. You may not reproduce, distribute or exploit the contents in any
form without written permission by copyright owner. Copyright infringers may face civil and criminal liability
Total Marks : 200.00
Prelims Test 23 - GS Test 16
( Insta Prelims Test Series 2024 7.0 ) Mark Scored : 86.90

Answer Justification :

The National Investigation Agency (NIA) was constituted in 2009 under the provisions of the
National Investigation Agency Act, 2008 (NIA Act). It is the central counter-terrorism law
enforcement agency in the country.

Hence, both the statements are correct.

29. Consider the following statements regarding CAG of India:


1. He holds his office till the pleasure of the president as he is appointed by him.
2. His salary and other service conditions are determined by the President of India.
3. He is not eligible for further office, either under the Government of India or of any state, 7 77 he
after
2
ceases to hold his office. 63
560
How many of the above statements are correct? - 9
m
A. Only one
il.co
B. Only two
m a
C. All three
4 ;g
D. None 6
5 &#
Your Answer : A va9
ta
Correct Answer : A
i vas
tisr
u
hr
Answer Justification :
- s
t a vamade the following provisions to safeguard and ensure the independence
The Constitution has
of CAG: as
S riv
1. Hetiis provided with the security of tenure. He can be removed by the president only in
ru
Shaccordance with the procedure mentioned in the Constitution. Thus, he does not hold his
office till the pleasure of the president, though he is appointed by him.

Hence, statement 1 is incorrect.

2. He is not eligible for further office, either under the Government of India or of any
state, after he ceases to hold his office.

Hence, statement 3 is correct.

3. His salary and other service conditions are determined by the Parliament. His salary
is equal to that of a judge of the Supreme Court.

Hence, statement 2 is incorrect.

4. Neither his salary nor his rights in respect of leave of absence, pension or age of retirement

instacourses.insightsonindia.com 25
© Insights Active Learning | All rights reserved - 18107. You may not reproduce, distribute or exploit the contents in any
form without written permission by copyright owner. Copyright infringers may face civil and criminal liability
Total Marks : 200.00
Prelims Test 23 - GS Test 16
( Insta Prelims Test Series 2024 7.0 ) Mark Scored : 86.90

can be altered to his disadvantage after his appointment.

5. The conditions of service of persons serving in the Indian Audit and Accounts Department and
the administrative powers of the CAG are prescribed by the president after consultation with
the CAG.

6. The administrative expenses of the office of the CAG, including all salaries, allowances and
pensions of persons serving in that office are charged upon the Consolidated Fund of India.
Thus, they are not subject to the vote of Parliament.

Further, no minister can represent the CAG in Parliament (both Houses) and no minister can be
called upon to take any responsibility for any actions done by him.

30. Consider the following statements :


7 77
Statement I : 3 2
Chief Election Commissioner and Other Election Commissioners (Appointment, 6 06
Conditions of Service and Term of Office) Act, 2023 aimed to shield these
- 95 officials
om
from civil or criminal proceedings related to their official functions.
Statement II :
l .c
m ai
This act safeguards CEC and ECs from legal proceedings related to actions taken
g
4; out in the discharge of official
during their tenure, provided such actions were carried
duties. # 6
&
a 95
tav of the above statements?
Which one of the following is correct in respect
s
A. Both Statement-I and Statement-II
r i va are correct and Statement-II is the correct explanation
for Statement-I
u tis
B. Both Statement-I and
shrStatement-II are correct and Statement-II is not the correct
-
explanation for Statement-I
acorrect
ta
C. Statement-I v
is but Statement-II is incorrect
s
a is incorrect but Statement-II is correct
riv
D. Statement-I
i S
r utAnswer : D
Your
ShCorrect Answer : A
Answer Justification :

Chief Election Commissioner and Other Election Commissioners (Appointment, Conditions of


Service and Term of Office) Act, 2023 safeguards CEC and ECs from legal proceedings related to
actions taken during their tenure, provided such actions were carried out in the discharge of
official duties.

Hence, statement II is correct.

This amendment aimed to shield these officials from civil or criminal proceedings related to
their official functions.

Hence, statement I is correct and option (a) is correct answer.

instacourses.insightsonindia.com 26
© Insights Active Learning | All rights reserved - 18107. You may not reproduce, distribute or exploit the contents in any
form without written permission by copyright owner. Copyright infringers may face civil and criminal liability
Total Marks : 200.00
Prelims Test 23 - GS Test 16
( Insta Prelims Test Series 2024 7.0 ) Mark Scored : 86.90

31. Consider the following statements regarding Tribunals in India:


1. The National Green Tribunal was established as the first Tribunal in India.
2. The Second Administrative Reforms Commission recommended that the central government
should set up Civil Services Tribunals at the national level and state levels.
3. The Sixth Law Commission (1974), recommend setting up a separate high-powered tribunal and
commission for adjudication of matters in High Courts.

How many of the above statements are correct?


A. Only one
B. Only two
C. All three
D. None

Your Answer : D 7
Correct Answer : A 277
6 3
560
Answer Justification :
- 9
m
In 1941, the Income Tax Appellate Tribunal was established asl.the
i co first Tribunal in India.
The objective was to reduce the workload of courts, expeditem a
adjudication of disputes, and build
;g
expertise on tax matters within the Tribunal.
# 64
&
The National Green Tribunal is a statutory body
a95 in India that deals with expeditious disposal
tavand other natural resources. It was set up under
of cases related to environmental protection
s
the National Green Tribunal Act in
r i va2010.
u tis
hr
Hence, statement 1 is incorrect.
s
v a - Reforms Commission recommended that the central government
The First Administrative
sta Services Tribunals at the national level and state levels.
should set upaCivil
S riv
i
ut statement 2 is incorrect.
Hence,
r
ShThe Sixth Law Commission (1974), recommend setting up a separate high-powered tribunal
and commission for adjudication of matters in High Courts. This was aimed at reducing arrears
of cases in the High Courts.

Hence, statement 3 is correct.

32. Consider the following statements regarding Public Services in India:


1. The All-India Services Act of 1951 empowered the Central Government to make rules in
consultation with the state governments to govern the recruitment and service conditions of all-
India service members.
2. Members of these services are recruited and trained by the central government before being
assigned to different states for work.
3. They are members of various state cadres, with the Centre lacking its own cadre in this regard.
instacourses.insightsonindia.com 27
© Insights Active Learning | All rights reserved - 18107. You may not reproduce, distribute or exploit the contents in any
form without written permission by copyright owner. Copyright infringers may face civil and criminal liability
Total Marks : 200.00
Prelims Test 23 - GS Test 16
( Insta Prelims Test Series 2024 7.0 ) Mark Scored : 86.90

How many of the above statements are incorrect?


A. Only one
B. Only two
C. All three
D. None

Your Answer : A
Correct Answer : D

Answer Justification :
The All-India Services Act of 1951 empowered the Central Government to make rules
in consultation with the state governments to govern the recruitment and service
conditions of all-India service members.

7
277
6 3
Hence, statement 1 is correct.
560
- 9
m
l.co
ai central government before
Members of these services are recruited and trained by the
m
being assigned to different states for work. ;g
#64
5 &
va9
ta
Hence, statement 2 is correct.
i vas
tisr
u
shr
-
va of various state cadres, with the Centre lacking its own cadre in this
They are members
regard. sta
a
Sriv
t i
h ru
S Hence, statement 3 is correct.

They serve on deputation for the Central Government and then return to their respective
states when their term expires.

33. Which one of the following statements is incorrect regarding the disqualification of
legislators?

A. If a member of a house belonging to a political party voluntarily gives up his membership


of same political party, then he will be disqualified.
B. If a nominated member joins a party six months after he becomes a member of the
legislature, he will be disqualified.

instacourses.insightsonindia.com 28
© Insights Active Learning | All rights reserved - 18107. You may not reproduce, distribute or exploit the contents in any
form without written permission by copyright owner. Copyright infringers may face civil and criminal liability
Total Marks : 200.00
Prelims Test 23 - GS Test 16
( Insta Prelims Test Series 2024 7.0 ) Mark Scored : 86.90

C. The Chairman or the Speaker of the House takes the decision to disqualify a member.
D. If an independent candidate joins a party after the election, he will not be disqualified.

Your Answer : B
Correct Answer : D

Answer Justification :

When can a legislator be disqualified?

a. If a member of a house belonging to a political party:

7
277
Voluntarily gives up membership of his political party, or 6 3
560
- 9
m
Hence, option (a) is correct. il.co
m a
4 ;g
6
&#
95political party, or does not vote in the House at
Votes contrary to a direction issued by his
a
all, when such a direction is issued.ta v
However, a member shall not be disqualified if he has
as
s r iv or is condoned by the party within 15 days from such
taken prior permission of his party,
voting or abstention. ti u
shr
-
t a va
i v as
b. If an independent candidate joins a party after the election.
r
t iS
ru
ShHence, option (d) is incorrect.

c. If a nominated member joins a party six months after he becomes a member of the
legislature.

Hence, option (b) is correct.

instacourses.insightsonindia.com 29
© Insights Active Learning | All rights reserved - 18107. You may not reproduce, distribute or exploit the contents in any
form without written permission by copyright owner. Copyright infringers may face civil and criminal liability
Total Marks : 200.00
Prelims Test 23 - GS Test 16
( Insta Prelims Test Series 2024 7.0 ) Mark Scored : 86.90

The Chairman or the Speaker of the House takes the decision to disqualify a member.

Hence, option (c) is correct.

34. Consider the following statements regarding the Political Party:


1. If a political party secures six percent of valid votes polled in any four or more states at a
general election to the Lok Sabha, it will get the status of state political party.
2. If a political party is recognised as a state party in four states, it will get the status of national
political party.

Which of the above statements is/are correct?


7
A. 1 only
277
B. 2 only 6 3
C. Both 1 and 2 560
- 9
D. Neither 1 nor 2
m
il.co
Your Answer : C
m a
Correct Answer : B 4 ;g
6
5 &#
a9
Answer Justification :
ta v
If a political party secures six percent a ofsvalid votes polled in any four or more states at a
i v
general election to the Lok Sabha,
tisrit will get the status of national political party.
h ru
- s sixper cent of the valid votes polled in the state at a general election
If a political party secures
a
st av
to the legislative assembly of the state concerned; and, in addition, it wins 2 seats in the

iva
assembly of the state concerned, then it will get the status of state party.
r
t
Hence, i Sstatement 1 is incorrect.
ru
ShIf a political party is recognised as a state party in four states, it will get the status of national
political party.

Hence, statement 2 is correct.

35. Consider the followings:


1. National Commission for Women
2. National Commission for Minorities
3. National Commission for BCs
4. National Commission for SCs

How many of the above are the constitutional bodies?


A. Only one

instacourses.insightsonindia.com 30
© Insights Active Learning | All rights reserved - 18107. You may not reproduce, distribute or exploit the contents in any
form without written permission by copyright owner. Copyright infringers may face civil and criminal liability
Total Marks : 200.00
Prelims Test 23 - GS Test 16
( Insta Prelims Test Series 2024 7.0 ) Mark Scored : 86.90

B. Only two
C. All three
D. None

Your Answer : B
Correct Answer : B

Answer Justification :

The National Commission for Scheduled Castes (SCs) is a constitutional body in the
sense that it is directly established by Article 338 of the Constitution.

On the other hand, the other national commissions like the National Commission for Women
(1992), the National Commission for Minorities (1993), the National Human Rights
Commission (1993) and the National Commission for Protection of Child Rights (2007) are 7
7
statutory bodies in the sense that they are established by acts of the Parliament. 27
63
560
- 9
m
102nd Constitution Amendment Act, 2018 provides constitutional
i l .co status to the
National Commission for Backward Classes (NCBC).
m a
4;g
Hence, option (b) is correct. 6
5 &#
va9
st a
va
36. Consider the following statements
i sri regarding Central Bureau of Investigation (CBI):
1. CBI has been constituted mainly
r ut for investigation of incidents of terrorist attacks, funding of
h
- s Act (2013) stated that the Central Government shall appoint the
terrorism and other terror related crime.
a
2. The Lokpal and Lokayuktas
v
tathe
Director of CBIson recommendation of a three-member committee consisting of the Prime
ivaLeader of Opposition in the Lok Sabha and the Speaker of Lok Sabha.
Minister, rthe
t iS
ru of the statements given above is/are not correct?
ShWhich
A. 1 only
B. 2 only
C. Both 1 and 2
D. Neither 1 nor 2

Your Answer : C
Correct Answer : C

Answer Justification :

The establishment of the CBI was recommended by the Santhanam Committee on


Prevention of Corruption (1962–1964).

Hence, statement 1 is correct.

instacourses.insightsonindia.com 31
© Insights Active Learning | All rights reserved - 18107. You may not reproduce, distribute or exploit the contents in any
form without written permission by copyright owner. Copyright infringers may face civil and criminal liability
Total Marks : 200.00
Prelims Test 23 - GS Test 16
( Insta Prelims Test Series 2024 7.0 ) Mark Scored : 86.90

There is a difference between the nature of cases investigated by the National Investigation
Agency (NIA) and the CBI. The NIA has been constituted after the Mumbai terror attack
in 2008 mainly for investigation of incidents of terrorist attacks, funding of terrorism
and other terror related crime, whereas the CBI investigates crime of corruption, economic
offences and serious and organized crime other than terrorism.

Hence, statement 2 is incorrect.

The Lokpal and Lokayuktas Act (2013) amended the Delhi Special Police Establishment Act
(1946) and made the following changes with respect to the composition of the CBI:

The Central Government shall appoint the Director of CBI on the recommendation of a
three-member committee consisting of the Prime Minister as Chairperson, the Leader of
Opposition in the Lok Sabha and the Chief Justice of India or Judge of the Supreme Court
nominated by him.
7
277
Hence, statement 3 is incorrect. 6 3
560
- 9
m
37. With reference to the National Disaster Management Authority .co
il (NDMA), consider the
a
following statements:
4 ;gm
6 of the National Disaster Management
1. Disaster Management Act, 2005 provided for the creation
#
Authority (NDMA). &
2. NDMA is the apex body for disaster managementa 95that works under the administrative control of
v
taand
s
the Union Ministry of Environment, Forest
a other Climate Change.
r
3. The NDMA consists of a chairperson i v
and members, not exceeding two.
u tis
How many of the above s h
statements
rare correct?
a -
av
A. Only one
B. Only twost
C. All r iva
three
S
D.tiNone
r u
Sh
Your Answer : A
Correct Answer : A

Answer Justification :

Disaster Management Act, 2005 provided for the creation of the National Disaster
Management Authority (NDMA) to spearhead and implement a holistic and integrated
approach to disaster management in the country. Initially, the NDMA was constituted in 2005 by
an Executive Order of the Government of India. Subsequently, the NDMA was notified in 2006
under the provisions of the Act.

Hence, statement 1 is correct.

The NDMA consists of a chairperson and other members, not exceeding nine. The Prime

instacourses.insightsonindia.com 32
© Insights Active Learning | All rights reserved - 18107. You may not reproduce, distribute or exploit the contents in any
form without written permission by copyright owner. Copyright infringers may face civil and criminal liability
Total Marks : 200.00
Prelims Test 23 - GS Test 16
( Insta Prelims Test Series 2024 7.0 ) Mark Scored : 86.90

Minister is the ex-officio chairperson of the NDMA. The other members are nominated by the
chairperson of the NDMA. The chairperson of the NDMA designates one of the members as the
vice-chairperson of the NDMA. The vice- chairperson has the status of a Cabinet Minister while
the other members have the status of a Minister of State.

Hence, statement 3 is incorrect.

The NDMA is the apex body for disaster management in the country. It works under the
administrative control of the Union Ministry of Home Affairs.

Hence, statement 2 is incorrect.

38. With reference to the Co-operative societies, consider the following statements:
7
77of
1. The state legislature may make provisions for the incorporation, regulation and winding-up
2
co-operative societies. 63
2. Right to form a co-operative society is a legal right in India. 60 5
-9
Which of the above statements is/are incorrect?
.c om
A. 1 only ai l
B. 2 only ;g m
4
C. Both 1 and 2
& #6
D. Neither 1 nor 2 5
va9
sta
Your Answer : D
va
Correct Answer : B i sri
t
h ru
Answer Justification-: s
t a va
Incorporation
i v as of Co-operative Societies: The state legislature may make provisions for
i Sr
the incorporation, regulation and winding-up of co-operative societies based on the principles of
ut formation, democratic member control, member- economic participation and
voluntary
r
h
S autonomous functioning.
Hence, statement 1 is correct.

The 97th Constitutional Amendment Act of 2011 gave a constitutional status and protection to
co-operative societies. In this context, it made the following three changes in the constitution:

It made the right to form co-operative societies a fundamental right (Article 19)

Hence, statement 2 is incorrect.

It included a new Directive Principle of State Policy on promotion of co-operative societies

instacourses.insightsonindia.com 33
© Insights Active Learning | All rights reserved - 18107. You may not reproduce, distribute or exploit the contents in any
form without written permission by copyright owner. Copyright infringers may face civil and criminal liability
Total Marks : 200.00
Prelims Test 23 - GS Test 16
( Insta Prelims Test Series 2024 7.0 ) Mark Scored : 86.90

(Article 43-B2 ).
It added a new Part IX-B in the Constitution which is entitled “The Co-operative Societies”
(Articles 243-ZH to 243-ZT).

39. Consider the following statements regarding the Notified Area Committee:
1. Since it is established by a notification in the government gazette, it is called as notified area
committee.
2. It is an entirely nominated body, that is, all the members of a notified area committee including
the chairman are nominated by the state government.

Which of the above statements is/are correct?


A. 1 only
B. 2 only
7
77
C. Both 1 and 2
D. Neither 1 nor 2 3 2
6
560
Your Answer : A - 9
Correct Answer : C m
il.co
Answer Justification : m a
4 ;g
6
Notified Area Committee
5 &#
v a9
A notified area committee is created for the
s ta administration of two types of areas–a fast
a
iv and a town which does not yet fulfil all the conditions
developing town due to industrialisation,
sr
i a municipality, but which otherwise is considered important by
uitt is established by a notification in the government
necessary for the constitution of
the state government. Sinceh r
gazette, it is called -
as
snotified area committee.
v a
a sta 1 is correct.
riv
Hence, statement
i S
ut it functions within the framework of the State Municipal Act, only those provisions of the
Though
r
Shact apply to it which are notified in the government gazette by which it is created. It may also
be entrusted to exercise powers under any other act. Its powers are almost equivalent to those
of a municipality. But unlike the municipality, it is an entirely nominated body, that is, all
the members of a notified area committee including the chairman are nominated by
the state government. Thus, it is neither an elected body nor a statutory body.

Hence, statement 2 is correct.

40. Consider the following statements regarding the Attorney General (AG) of India:
1. He holds office for a period of six years or upto the age of 65 years, whichever is earlier.
2. He can be removed by the president on the basis of a resolution passed to that effect by both
the Houses of Parliament with special majority, either on the ground of proved mis behavior or
incapacity.

instacourses.insightsonindia.com 34
© Insights Active Learning | All rights reserved - 18107. You may not reproduce, distribute or exploit the contents in any
form without written permission by copyright owner. Copyright infringers may face civil and criminal liability
Total Marks : 200.00
Prelims Test 23 - GS Test 16
( Insta Prelims Test Series 2024 7.0 ) Mark Scored : 86.90

3. His remuneration is decided by the Parliament of India.

How many of the above statements are correct?


A. Only one
B. Only two
C. All three
D. None

Your Answer : D
Correct Answer : D

Answer Justification :

The term of office of the AG is not fixed by the Constitution.


7
Hence, statement 1 is incorrect. 277
3
06
56his removal.
Further, the Constitution does not contain the procedure and grounds for
-9
Hence, statement 2 is incorrect.
.c om
ail
;gm that he may be removed by the
He holds office during the pleasure of the president. This means
4
# 6
president at any time. He may also quit his office by submitting his resignation to the president.
&
Conventionally, he resigns when the government5(council of ministers) resigns or is replaced, as
a9
v of the AG is not fixed by the Constitution. He
he is appointed on its advice. The remuneration
receives such remuneration as the a sta
president may determine.
sr iv
u
Hence, statement 3 is incorrect. ti
shr
a -
stav
a
riv following statements:
41. Consider the
S
ti Shakti is aimed at strengthening interventions for women empowerment.
1. Mission
ru National Creche Scheme has sub-schemes called 'Sambal' and 'Samarthya'.
2. hThe
S
3. There is no provision for gap funding under the Samarthya Scheme.

How many of the statements given above are correct?


A. Only one
B. Only two
C. All three
D. None

Your Answer :
Correct Answer : A

Answer Justification :

Ministry of Women and Child Development has issued detailed guidelines for ‘Mission Shakti'

instacourses.insightsonindia.com 35
© Insights Active Learning | All rights reserved - 18107. You may not reproduce, distribute or exploit the contents in any
form without written permission by copyright owner. Copyright infringers may face civil and criminal liability
Total Marks : 200.00
Prelims Test 23 - GS Test 16
( Insta Prelims Test Series 2024 7.0 ) Mark Scored : 86.90

scheme.

Statement 1 is correct: The Government of India has launched 'Mission Shakti' - an


integrated women empowerment programme as umbrella scheme for the safety, security and
empowerment of women for implementation during the 15th Finance Commission period
202l-22 to 2025-26.

Statement 2 is not correct: Mission Shakti’ has two sub-schemes - 'Sambal' and 'Samarthya'.
While the "Sambal" sub-scheme is for safety and security of women, the "Samarthya" sub-
scheme is for empowerment of women.

Statement 3 is not correct: The components of 'Samarthya' sub-scheme consist of erstwhile


schemes of Ujjwala, Swadhar Greh and Working Women Hostel have been included with
modifications. A new component of Gap Funding for Economic Empowerment has also been
added in the Samarthya Scheme.
7
277
6 3
560
42. Consider the following statements: - 9
m
.co
1. The Rare Earth Elements are used to make permanent magnets and used in span sectors like
imaging and aerospace. ai l
2. Lanthanum, Neodymium and Zirconium are crucial Rare Earth ;g m Elements none of which are
4
found in India.
& #6
9 5
3. India is not a member of the Mineral Security Partnership that aims to conserve the Rare Earth
Elements. va
asta
r v is/are not correct?
iabove
How many of the statements given
i s
A. Only one r ut
B. Only two - sh
C. All three ava
D. None as
t
S riv
i
utAnswer : B
Your
r
ShCorrect Answer : B
Answer Justification :

Minerals like Cobalt, Nickel, and Lithium are required for batteries used in electric vehicles. Rare
Earth Elements/REEs are an essential although often tiny component of mobile phones, electric
and hybrid vehicles, semiconductors, flatscreen TVs and monitors, and high-end electronics.

Statement 1 is correct: The Rare Earth Elements are key components in many electronic
devices and whose industrial applications span sectors like imaging, aerospace, and defence.

Most important end-use of REEs was to make permanent magnets, which have essential use in
modern electronic devices like smartphones, computers, jet aircraft, and other products.

The REEs are lanthanum, cerium, praseodymium, neodymium, yttrium, hafnium, tantalum,

instacourses.insightsonindia.com 36
© Insights Active Learning | All rights reserved - 18107. You may not reproduce, distribute or exploit the contents in any
form without written permission by copyright owner. Copyright infringers may face civil and criminal liability
Total Marks : 200.00
Prelims Test 23 - GS Test 16
( Insta Prelims Test Series 2024 7.0 ) Mark Scored : 86.90

niobium, zirconium, and scandium.

Statement 2 is not correct: Some REEs are available in India such as Lanthanum, Cerium,
Neodymium, Praseodymium and Samarium, etc.

Others such as Dysprosium, Terbium, and Europium, which are classified as HREEs, are not
available in Indian deposits in extractable quantities.

Statement 3 is not correct: India has become 14th member of the Mineral Security
Partnership (MSP) in June 2023.

The other member countries are United States, Australia, Canada, Finland, France, Germany,
Italy, Japan, Norway, the Republic of Korea, Sweden, the United Kingdom and the European
Commission.

7
277
3
43. Consider the following statements: 6 06
5
- 9 the loss of
1. Environmental inhibitors improve the production efficiency of crops while limiting
nitrogen from the pastures.
.c om
ai l
2. An inadvertent presence of environmental inhibitors in food commodities creates trade
disruption if standards are not established. g m ;
#64
&
Which of the statements given above is/are correct?
a95
tav
A. 1 only
B. 2 only s
r i va
tis
C. Both 1 and 2
D. Neither 1 nor 2 u
shr
-
Your Answer : B va
Correct Answera st: aC
S riv
i
ut Justification :
Answer
r
Sh
Option (c) is correct: Food and Agriculture Organisation/ FAO has released a report on Food
safety implications from the use of environmental inhibitors in agrifood systems, which was
developed within the activities of FAO’s Food Safety Foresight Programme.

Environmental inhibitors are one of the approaches used to minimize the harmful effects
of agrifood systems on the environment.
Not only can they reduce greenhouse gas emissions, such as methane from ruminants or
limit the loss of nitrogen from cultivated fields and pastures, but also improve the
efficiency of livestock and crop production.”
However, an inadvertent presence of environmental inhibitors in food commodities can
raise health concerns as well as trade disruption if standards or appropriate measures are
not established.
The food safety risks associated with these substances can be challenging to assess and
manage due to the lack of internationally harmonized maximum residue limits (MRLs),

instacourses.insightsonindia.com 37
© Insights Active Learning | All rights reserved - 18107. You may not reproduce, distribute or exploit the contents in any
form without written permission by copyright owner. Copyright infringers may face civil and criminal liability
Total Marks : 200.00
Prelims Test 23 - GS Test 16
( Insta Prelims Test Series 2024 7.0 ) Mark Scored : 86.90

agreed definition for environmental inhibitors and insufficient safety information for some
compounds.

44. Consider the following statements:


Polar stratospheric clouds (PSCs):
1. Provide surfaces upon which heterogeneous chemical reactions take place.
2. Play a central role in the formation of the ozone hole.
3. Include mother-of-pearl clouds that are composed of ice crystals.

How many of the statements given above are correct?


A. Only one
B. Only two
C. All three
D. None 7
277
6 3
Your Answer : C
560
Correct Answer : C
- 9
m
Answer Justification : il.co
m a
;g a central role in the formation of
Option (c) is correct: Polar stratospheric clouds (PSCs)4play
the ozone hole in the Antarctic and Arctic. #6 &
a 95
s tav
PSCs provide surfaces upon which heterogeneous chemical reactions take place.
i va
These reactions lead to the production of free radicals of chlorine in the stratosphere
r
tis molecules.
which directly destroy ozone
u
PSCs form poleward ofr about 60°S latitude in the altitude range 10 km to 25 km during
- shspring.
the winter and early
vaclassified into Types I and II according to their particle size and formation
The cloudstaare
v as
temperature.
i
i Sr II clouds, also known as nacreous or mother-of-pearl clouds, are composed of ice
Type
u t
r crystals and form when temperatures are below the ice frost point (typically below
Sh −83°C).
The Type I PSCs are optically much thinner than the Type II clouds, and have a formation
threshold temperature 5 to 8°C above the frost point. These clouds consist mainly of
hydrated droplets of nitric acid and sulphuric acid.

45. With reference to the Sand Mining, consider the following statements:
1. It can be extracted through marine dredging which leads to the salination of aquifers.
2. Sand is categorised as a Major mineral in India under the Mines and Minerals (Development &
Regulation) Act, 1957.
3. The control of illegal mining in India comes under the exclusive purview of the Central
Government.

How many of the statements given above are correct?

instacourses.insightsonindia.com 38
© Insights Active Learning | All rights reserved - 18107. You may not reproduce, distribute or exploit the contents in any
form without written permission by copyright owner. Copyright infringers may face civil and criminal liability
Total Marks : 200.00
Prelims Test 23 - GS Test 16
( Insta Prelims Test Series 2024 7.0 ) Mark Scored : 86.90

A. Only one
B. Only two
C. All three
D. None

Your Answer : A
Correct Answer : A

Answer Justification :

Statement 1 is correct: Sand mining from rivers and marine ecosystems can lead to erosion,
salination of aquifers, loss of protection against storm surges and impacts on biodiversity, which
pose a threat to livelihoods through, among other things, water supply, food production,
fisheries, or to the tourism industry.
77
27
Around 6 billion tonnes of marine sand is being dug up each year by the marine dredging
3
industry. 06
9 56
Statement 2 is not correct: Sand is a minor mineral under Section 3(e) of the Mines and -
Minerals (Development & Regulation) Act, 1957 (MMDR Act). Section .c o15mof the MMDR Act
empowers the State Governments for making rules for regulating
l
ai the grant of quarry leases,
m
;g minerals and for purposes
mining leases or other mineral concessions in respect of4minor
connected therewith.
& #6
a95
s tav 23C of the MMDR Act empowers the State
Statement 3 is not correct: Further, Section

r i va illegal mining, transportation and storage of minerals


Governments to make rules for preventing

ts
and for the purposes connecteditherewith.
u
shr comes under the legislative and administrative purview of the
a-
Hence, control of illegal mining
State Governments. v
a sta
Sriv
i
r ut
Sh
46. Consider the following statements:
1. The Geological Survey of India was originally set up to find coal deposits for the Railways.
2. The Ministry of Mines has launched the National Geoscience Data Repository (NGDR) Portal.
3. The Geological Survey of India is the nodal agency for implementing the National Geoscience
Data Repository Initiative.

How many of the statements given above are correct?


A. Only one
B. Only two
C. All three
D. None

Your Answer : C

instacourses.insightsonindia.com 39
© Insights Active Learning | All rights reserved - 18107. You may not reproduce, distribute or exploit the contents in any
form without written permission by copyright owner. Copyright infringers may face civil and criminal liability
Total Marks : 200.00
Prelims Test 23 - GS Test 16
( Insta Prelims Test Series 2024 7.0 ) Mark Scored : 86.90

Correct Answer : C

Answer Justification :

Statement 1 is correct: The Geological Survey of India (GSI) was set up in 1851 primarily to
find coal deposits for the Railways.

Over the years, GSI has not only grown into a repository of geo-science information required in
various fields in the country but has also attained the status of a geo-scientific organisation of
international repute.

Statement 2 is correct: Ministry of Mines is to launch the National Geoscience Data


Repository (NGDR) Portal on 19th December 2023.

NGDR is a comprehensive online platform for accessing, sharing, and analyzing geospatial
7
information across the nation. 77 2
3
06
56 by Ministry
National Geoscience Data Repository (NGDR) is a flagship initiative conceptualised
9
-
of Mines as a part of National Mineral Exploration Policy (NMEP), 2016 for hosting all exploration
mto expedite, enhance
related geoscientific data for dissemination to all the stakeholders sooas
.c
and facilitate the exploration coverage of the country. ail
4 ;gm
Statement 3 is correct: Geological Survey of India is6selected as the nodal agency for the
implementation of NGDR.
5 &#
va9
st a
va
i sri
ut
47. The Project PRAYAS has been recently launched in India for-
r
- sh
A. Promoting theainternational migration cycle for youth and Professionals.
B. Promoting staanv emergency response and healthcare system package.
a
riv financial risk protection against catastrophic health expenditure.
C. Providing
S
D. iProviding a step forward in enhancing India’s maritime security infrastructure.
r ut
Sh Your Answer :
Correct Answer : A

Answer Justification :

Option (a) is correct: The launch of Project PRAYAS (Promoting Regular & Assisted Migration
for Youth and Skilled Professionals).

The project is a joint collaboration between the IOM India and Indian Council of World
Affairs (ICWA).
Project PRAYAS not only aligns with Goverment of India's prioities for the promotion of
safe and orderly migration, but also aligns with Goal 10.7 of the 2030 Agenda for
Sustainable Development Goals (SDGs) to facilitate orderly, safe, regular, and responsible
migration and mobility of people.
The project also adheres to the objectives of Global Compact for Safe, Orderly and
instacourses.insightsonindia.com 40
© Insights Active Learning | All rights reserved - 18107. You may not reproduce, distribute or exploit the contents in any
form without written permission by copyright owner. Copyright infringers may face civil and criminal liability
Total Marks : 200.00
Prelims Test 23 - GS Test 16
( Insta Prelims Test Series 2024 7.0 ) Mark Scored : 86.90

Regular Migration (GCM) and the Migration Governance Framework (MiGOF).

48. Consider the following statements:


Statement-I:
Fly ash is formed from the mineral matter in coal consisting of a small amount of
carbon that remains from incomplete combustion.
Statement-II:
A pozzolanic material is a form of fly ash which hardens with water but only after
activation with an alkaline substance.

Which one of the following is correct in respect of the above statements?


A. Both Statement-I and Statement-II are correct and Statement-II is the correct explanation
for Statement-I
B. Both Statement-I and Statement-II are correct and Statement-II is not the correct
7
explanation for Statement-I
2 77
C. Statement-I is correct but Statement-II is incorrect
0 63
D. Statement-l is incorrect but Statement-II is correct 56
-9
Your Answer : B
.c om
Correct Answer : B
m ail
4 ;g
6
&#
Answer Justification :
95
t a vacalled fly ash because it is transported from the
Fly ash is the finest of coal ash particles. It is
combustion chamber by exhaust gases.
ivas
s r
Statement-I is correct: Flyu ti is the fine powder formed from the mineral matter in coal,
ash
r
- sh
consisting of the noncombustible matter in coal and a small amount of carbon that remains
a
tav
from incomplete combustion.
a s
Properties
S riofv fly ash vary significantly with coal composition and plant operating conditions. Fly
ireferred to as either cementations or pozzolanic.
ut
ash is
r
ShStatement-II is correct: A cementations material is one that hardens when mixed with water.
A pozzolanic material will also harden with water but only after activation with an alkaline
substance such as lime.

These cementations and pozzolanic properties make some fly ashes useful for cement
replacement in concrete and many other building applications.

Fly ash is used in concrete and cement products, road base, oil stabilizer, clean fill, filler in
asphalt, metal recovery, and mineral filler.

49. Consider the following statements regarding Papua New Guinea:


1. It is geographically positioned entirely in the Southern Hemisphere.
2. It is bounded by the Pacific Ocean and Torres Strait.

instacourses.insightsonindia.com 41
© Insights Active Learning | All rights reserved - 18107. You may not reproduce, distribute or exploit the contents in any
form without written permission by copyright owner. Copyright infringers may face civil and criminal liability
Total Marks : 200.00
Prelims Test 23 - GS Test 16
( Insta Prelims Test Series 2024 7.0 ) Mark Scored : 86.90

3. Its neighbours include Indonesia, Australia and Solomon Islands.

How many of the statements given above is/are correct?


A. Only one
B. Only two
C. All three
D. None

Your Answer : B
Correct Answer : B

Answer Justification :

Papua New Guinea is an island nation in Oceania located in southwestern Pacific Ocean.

It occupies the eastern half of New Guinea (the world’s 2nd largest island) and includes27
77
3
numerous offshore islands. 06
9 56
Statement 1 is not correct: Papua New Guinea is geographically positioned both in the -
m
Southern and Eastern hemispheres of the Earth. .co il
m a
;g Bismarck Sea, Solomon Sea, Coral
Statement 2 is correct: It is bounded by the Pacific Ocean,
4
Sea, Torres Strait and Gulf of Papua. #6
9 5&
a
av Indonesia to the west; Australia to the south and
Statement 3 is correct: Its neighbours are:
Solomon Islands to the south-east. ast
sr iv
i
r ut
- sh
Debt Report 2023”?t a va
50. Which one of the following Organisations has recently released the “International

i v as
Sr
A.tiThe World Bank
ru
Sh B. The World Economic Forum
C. The International Monetary Fund
D. The United Nations Development Programme

Your Answer : C
Correct Answer : A

Answer Justification :

Option (a) is correct: Amid the biggest surge in global interest rates in four decades,
developing countries spent a record $443.5 billion to service their external public and publicly
guaranteed debt in 2022, the World Bank’s latest International Debt Report shows.

The increase in costs shifted scarce resources away from critical needs such as health,
education, and the environment.

instacourses.insightsonindia.com 42
© Insights Active Learning | All rights reserved - 18107. You may not reproduce, distribute or exploit the contents in any
form without written permission by copyright owner. Copyright infringers may face civil and criminal liability
Total Marks : 200.00
Prelims Test 23 - GS Test 16
( Insta Prelims Test Series 2024 7.0 ) Mark Scored : 86.90

The International Debt Report (IDR) is a longstanding annual publication of the World
Bank featuring external debt statistics and analysis for the 122 countries that report to
the World Bank Debtor Reporting System.

51. Consider the following statements regarding Kelps:


1. They are found along rocky intertidal and shallow coastal areas.
2. They dominate autotrophic biomass in temperate and Arctic regions.
3. They are environmentally sensitive acting as bio-indicators of the reef ecosystems.

How many of the statements given above is/are correct?


A. Only one
B. Only two
7
77
C. All three
D. None 3 2
6
560
Your Answer : C
- 9
Correct Answer : C m
il.co
Answer Justification : m a
4 ;g
Statement 1 is correct: Kelps are large, brown algae& #6found along rocky intertidal and shallow
5
coastal areas globally.
va9
s ta
Statement 2 is correct: They dominate a
iv autotrophic biomass in temperate and Arctic regions,
sr
where they form extensive stands
u ti forests.
or

shr
Statement 3 is correct:
a - Kelp forests are sensitive to environmental conditions, and are
v
ta integrity of temperate reef ecosystems.
bioindicators of the
s
va
ridepends
As growthS on interactions among temperature, nutrient availability, and light, kelp
i are threatened by declining water quality and climate change.
t
ru
forests
Sh
Unlike salt-marsh plants, mangroves, and seagrasses, kelp species have no belowground
biomass, as kelp plants are anchored to the surface of hard substrate by holdfasts.

52. Which one of the following statements best describes the term “Red Tide”?

A. It refers to the oil spill floating on the ocean water that destroys insulating ability of fur-
bearing mammals.
B. It refers to an event that occurs on the coastline when algae which is a plant-like
organism grows out of control.
C. It refers to the long-period waves that move through the ocean in response to the forces
exerted by the moon and sun.
D. It refers to the massive movement of water that specifically can be converted into a

instacourses.insightsonindia.com 43
© Insights Active Learning | All rights reserved - 18107. You may not reproduce, distribute or exploit the contents in any
form without written permission by copyright owner. Copyright infringers may face civil and criminal liability
Total Marks : 200.00
Prelims Test 23 - GS Test 16
( Insta Prelims Test Series 2024 7.0 ) Mark Scored : 86.90

usable form of energy.

Your Answer : B
Correct Answer : B

Answer Justification :

Option (b) is correct: A red tide is an event that occurs on the coastline when algae—a plant-
like organism—grows out of control.

The name “red tide” comes from the fact that overgrown algae can cause the water to
change color. Red tides can be hazardous to human health and sea life.
Red tides are caused by algae, which are tiny, microscopic organisms that grow in the
water. Almost all bodies of water have some algae, but in a red tide, there is a lot more
algae in the water than usual. 7
2
In fact, the water changes color in a red tide because the population of algae living 7in7 the
3
water becomes so dense.
6 06
Red tides have been around since long before humans. However, certain 5human activities
are making them more frequent. -9
Red tides are sometimes also called harmful algal blooms. Some .c om
of the algae that causes
a i l
a red tide produce powerful toxins, which are harmful chemicals that can kill fish,
m
shellfish, mammals and birds. 4;g
& #6
9 5
53. Consider the following statements regarding
ta va the Noma Disease:
1. It is a gangrenous disease of mouth which
i vas has 90% mortality rate.
2. It has been listed as a neglected r
istropical disease by the World Health Organization.
u t
r
3. It affects children who arehmalnourished and infected with infectious diseases.
a -s
How many of the v
tastatements given above are correct?
s
iva
A. Only one
r
S
B. iOnly two
u t
h r All three
C.
S D. None
Your Answer :
Correct Answer : C

Answer Justification :

Option (c) is correct: The World Health Organization (WHO) on December 15, 2023 added
one of the world’s most underrecognised health challenges, noma, to its official list of neglected
tropical diseases (NTD).

Also known as cancrum oris or gangrenous stomatitis, it is a severe gangrenous disease


of the mouth and face with a mortality rate of approximately 90 per cent.
Noma primarily affects children who are malnourished, infected with infectious diseases,
living in extreme poverty with poor oral health, or who have weakened immune systems,

instacourses.insightsonindia.com 44
© Insights Active Learning | All rights reserved - 18107. You may not reproduce, distribute or exploit the contents in any
form without written permission by copyright owner. Copyright infringers may face civil and criminal liability
Total Marks : 200.00
Prelims Test 23 - GS Test 16
( Insta Prelims Test Series 2024 7.0 ) Mark Scored : 86.90

according to the WHO.


Because the disease has a high mortality rate, early detection is critical because therapy
is most effective in the early stages of the disease.
Its spread can be slowed significantly with basic hygiene, antibiotics, and nutritional
rehabilitation.

54. Consider the following statements:


The Kunming-Montreal Global Biodiversity Framework-
1. Was adopted during the fifteenth meeting of the Conference of the Parties or (COP 15).
2. Includes a global target to conserve at least 30% of the world’s land and seas by 2030.
3. Aims to protect the Earth’s ozone layer by phasing out the chemicals that deplete it.

Which of the statements given above is/are correct?


A. 1 only 7
B. 1 and 2 only 277
6 3
C. 2 and 3 only
560
D. 1, 2 and 3
- 9
m
Your Answer : C il.co
Correct Answer : B m a
4 ;g
6
Answer Justification :
5 &#
va9
Statement 1 is correct: The Kunming-Montreal
s ta Global Biodiversity Framework (GBF) was
adopted during the fifteenth meeting i va
of the Conference of the Parties (COP 15) following a four
s r
i process.
ut
year consultation and negotiation
h r
This historic Framework,- swhich supports the achievement of the Sustainable Development
Goals and buildsta vathe Convention’s previous Strategic Plans, sets out an ambitious pathway to
on
v as vision of a world living in harmony with nature by 2050.
reach the global
i
i Sr
t
h ru
Statement 2 is correct: The agreement’s 23 targets include a global target to conserve at
S least 30% of the world’s land, freshwater and seas by 2030, as well as a target to restore 30%
of degraded places, also by 2030.

Statement 3 is not correct: The Montreal Protocol (Not the Kunming-Montreal Global
Biodiversity Framework) on Substances that Deplete the Ozone Layer is a global agreement to
protect the Earth’s ozone layer by phasing out the chemicals that deplete it.

55. Consider the following statements:


1. An alternative investment includes stocks, bonds, and cash.
2. Transaction costs are typically low for an alternative asset.
3. Cryptocurrency is classified as an alternative investment.

How many of the statements given above is/are not correct?

instacourses.insightsonindia.com 45
© Insights Active Learning | All rights reserved - 18107. You may not reproduce, distribute or exploit the contents in any
form without written permission by copyright owner. Copyright infringers may face civil and criminal liability
Total Marks : 200.00
Prelims Test 23 - GS Test 16
( Insta Prelims Test Series 2024 7.0 ) Mark Scored : 86.90

A. Only one
B. Only two
C. All three
D. None

Your Answer : A
Correct Answer : A

Answer Justification :

Statement 1 is not correct: An alternative investment is a financial asset that does not fit
into the conventional equity/income/cash categories. Conventional categories include stocks,
bonds, and cash.

77
Private equity or venture capital, hedge funds, real property, commodities, and tangible assets
are all examples of alternative investments. 27
3
6 06
- 95
Statement 2 is correct: Although alternative assets may have high initial minimums and
upfront investment fees, transaction costs are typically lower than those of conventional assets
due to lower levels of turnover. .c om
ai l
;g
Statement 3 is correct: Common forms of alternative investmentsm include real estate,
4
commodities, cryptocurrency, and collectibles.
& #6
5
v a9
asta
56. Consider the following passage:riv
This is NASA’s Mission which u tiiss a part of a constellation of instruments measuring air
quality over the Northern s hrHemisphere. Its measurements from the geostationary
a - and clouds create a dataset that improves the prediction of
orbit of ozone, aerosols,
v
s a
air quality and tphysical effects on climate. It will measure atmospheric pollution
a of North America, and the Atlantic at high spatial resolution.
riv
covering most
i S
r ut one of the following Missions is best described in the passage given above?
Which
Sh A. HaloSat Mission
B. QUESST Mission
C. MOXIE Mission
D. TEMPO Mission

Your Answer :
Correct Answer : D

Answer Justification :

Option (d) is correct: The TEMPO (Tropospheric Emissions: Monitoring of Pollution (EVI-1))
Mission is part of a constellation of instruments measuring air quality over the Northern
Hemisphere.

Its measurements from geostationary orbit of ozone, aerosols, and clouds will create a

instacourses.insightsonindia.com 46
© Insights Active Learning | All rights reserved - 18107. You may not reproduce, distribute or exploit the contents in any
form without written permission by copyright owner. Copyright infringers may face civil and criminal liability
Total Marks : 200.00
Prelims Test 23 - GS Test 16
( Insta Prelims Test Series 2024 7.0 ) Mark Scored : 86.90

revolutionary dataset that provides understanding and improves prediction of air quality
and physical affects on climate.
Its objective is to measure pollution of North America, from Mexico City to the Canadian
oil sands, and from the Atlantic to the Pacific hourly and at high spatial resolution.
TEMPO spectroscopic measurements in the ultraviolet and visible provide a tropospheric
measurement suite that includes the key elements of tropospheric air pollution chemistry.

57. Consider the following statements regarding the Terms of Trade (TOT):
1. It is defined as the ratio between the index of export prices and the index of import prices.
2. It is independent of inflation and prices and can be calculated using implicit price deflators.

Which of the statements given above is/are correct?


A. 1 only
7
77
B. 2 only
C. Both 1 and 2 3 2
6
D. Neither 1 nor 2
560
- 9
Your Answer : B m
Correct Answer : A il.co
m a
4 ;g
Answer Justification : 6
5 &#
Statement 1 is correct: Terms of trade (TOT)
v a9represent the ratio between a country's export
a
stare defined as the value of a country's total exports
prices and its import prices. TOT indexes
minus total imports. i va
tisr
The ratio is calculated by h r u
dividing the price of the exports by the price of the imports and
multiplying the result
s
- 100.
a by
stav
If the exportva
i prices increase more than the import prices, a country has a positive terms of
rfor
trade, as
i S the same amount of exports, it can purchase more imports.
t
ru
ShStatement 2 is not correct: A TOT is dependent to some extent on exchange and inflation
rates and prices. A variety of other factors influence the TOT as well, and some are unique to
specific sectors and industries.

ToT can be calculated using so-called implicit price deflators. Gross Value Added (GVA) — the
value of all goods and services produced in an economy minus the value of their inputs
consumed during a given year, net of product taxes and subsidies — is estimated at both
current and constant prices.

58. Consider the following statements regarding the ‘Operation Prosperity Guardian’:
1. It is a military operation by a predominantly Western multi-national coalition.
2. It was formed to respond to the Houthi-led attacks on shipping in the Red Sea.

instacourses.insightsonindia.com 47
© Insights Active Learning | All rights reserved - 18107. You may not reproduce, distribute or exploit the contents in any
form without written permission by copyright owner. Copyright infringers may face civil and criminal liability
Total Marks : 200.00
Prelims Test 23 - GS Test 16
( Insta Prelims Test Series 2024 7.0 ) Mark Scored : 86.90

Which of the statements given above is/are correct?


A. 1 only
B. 2 only
C. Both 1 and 2
D. Neither 1 nor 2

Your Answer :
Correct Answer : C

Answer Justification :

Option (c) is correct: Maersk, one of the world’s largest shipping companies, is preparing to
resume its ships’ passage through the Red Sea/Gulf of Aden.

7
This follows the setting up of the multi-national security initiative Operation Prosperity Guardian
277
(OPG) to allow maritime ships passage through the critical Red Sea/Gulf of Aden that connects
3
Asia with Europe and the US. 06
9 56
-
The OPG is a US-led military operation by a predominantly Western multi-national coalition
m
.cino the Red Sea.
formed a few days ago to respond to Houthi-led attacks on shipping
il
m a
4 ;g
& #6
5
59. Iceland is an island nation which directly borders-
va9
A. The Norwegian Sea asta
B. The Baltic Sea sr iv
i
C. The Aegean Sea r ut
D. The Adriatic Sea- s
h
t a va
Your Answer a :s
A
r i v
CorrectSAnswer : A
i
r ut
ShAnswer Justification :
Option (a) is correct: Iceland is an island nation situated in the North Atlantic Ocean, directly
borders the Greenland Sea and the Norwegian Sea.

The Eastern Region of Iceland is known for its extensive fjords, which are longer and more
intricate than those in the northwest.
The landscape is a mix of mountains and coastal areas, with small fishing villages and a
sparse population.
The Central Highlands are the most remote part of Iceland, uninhabitable and accessible
only during the summer.

60. Consider the following statements regarding the Late Blight Disease:
1. It is a devastating disease of potatoes and tomatoes.

instacourses.insightsonindia.com 48
© Insights Active Learning | All rights reserved - 18107. You may not reproduce, distribute or exploit the contents in any
form without written permission by copyright owner. Copyright infringers may face civil and criminal liability
Total Marks : 200.00
Prelims Test 23 - GS Test 16
( Insta Prelims Test Series 2024 7.0 ) Mark Scored : 86.90

2. The organism that causes it is an oomycete related to diatoms.


3. India is the genetic centre of origin of the disease.

How many of the statements given above is/are correct?


A. Only one
B. Only two
C. All three
D. None

Your Answer :
Correct Answer : B

Answer Justification :

Statement 1 is correct: Late blight Disease is caused by the fungus like oomycete pathogen 7
277
Phytophthora infestans, is a devastating disease that can infect potato foliage and tubers at any
3
stage of crop development. 06
9 56
Statement 2 is correct:The organism that causes it, Phytophthora infestans, -
is an oomycete
om
related to diatoms and brown algae. l.c
m ai
Statement 3 is not correct: The origin of the 1845 epidemic4 ;g is not known with certainty but it
#6this time.
is likely that the disease did not occur in Europe before
&
5
va9and occurs in almost all regions where potatoes
ta
Late blight has subsequently spread worldwide
ssmall-scale
are grown, causing losses for large- and
i va growers alike.
is r
The genetic center of originrofutP. infestans has long been debated; originally it was assumed
-
that the pathogen originatedsh from the Andes along with its host plant, potato.
t a va
i v as
i Srthe following statements:
61. Considert
ru Carbon Credit Trading Scheme, 2023 was notified in India under the Energy Conservation
1. hThe
S Act, 2001.
2. The Bureau of Energy Efficiency is the administrator of Indian Carbon Market and the Green
House Gas emissions trajectory.
3. The Central Electricity Regulatory Commission is the regulator for the trading of carbon credit
certificates.

How many of the statements given above are correct?


A. Only one
B. Only two
C. All three
D. None

Your Answer : C
Correct Answer : C

instacourses.insightsonindia.com 49
© Insights Active Learning | All rights reserved - 18107. You may not reproduce, distribute or exploit the contents in any
form without written permission by copyright owner. Copyright infringers may face civil and criminal liability
Total Marks : 200.00
Prelims Test 23 - GS Test 16
( Insta Prelims Test Series 2024 7.0 ) Mark Scored : 86.90

Answer Justification :

Statement 1 is correct: India’s Carbon Credit Trading Scheme, 2023 (CCTS 2023) was
notified by the Government of India under the Energy Conservation Act, 2001, to develop the
country’s first-ever domestic carbon market.

The notification underlines the necessary framework and the roles of diverse stakeholders for
the development and functioning of the Indian Carbon Market (ICM).

Statement 2 is correct: The Bureau of Energy Efficiency (BEE) will be the administrator for
the ICM and will be responsible for the development of the GHG emissions trajectory and the
targets for the entities to be obligated under the notification.

Statement 3 is correct: The Central Electricity Regulatory Commission (CERC) will be the
regulator for the trading of carbon credit certificates.
7
277
3
6 06
62. The Madhukar Gupta Committee Report is sometimes talked about in9the 5 news in the
-
context of-
.c om
i l
aborder
A. Examining security mechanisms for Indian states sharing
;g m with Pakistan.
4
6 lives immigrants in India.
B. Examining national policy decisions that affect the
#
9 5& tribal concentrated areas.
C. Expanding welfare programmes and initiatives to
D. Constituting of the National Inspectionva
and Investigation Committee.
ta
i vas
isr
Your Answer : A
Correct Answer : A u t
shr
Answer Justificationv a -:
a sta
rivis correct: The Madhukar Gupta Committee submitted its report to the
Option (a)
S
i
ut
Government to strengthen border protection and address vulnerabilities in fencing along the
r
Sh
Indo-Pakistan border.

The Committee has given its recommendations broadly on the issues of Threats and
Border Protection, assessment of force level, deployment on the border, infrastructure
and technology issues for protection of border and administrative issues.
It has been decided to initiate action in light of the recommendations of the Committee in
consultation with the stakeholders.

63. With reference to Namdapha Flying Squirrel, consider the following statements:
1. It is a nocturnal species indigenous to Changlang district of Arunachal Pradesh.
2. It is also referred to as the Red Giant Flying Squirrel due to its red patagium.
3. It is protected under Schedule I of the Wildlife Protection (Amendment) Act 2022.

How many of the statements given above is/are not correct?

instacourses.insightsonindia.com 50
© Insights Active Learning | All rights reserved - 18107. You may not reproduce, distribute or exploit the contents in any
form without written permission by copyright owner. Copyright infringers may face civil and criminal liability
Total Marks : 200.00
Prelims Test 23 - GS Test 16
( Insta Prelims Test Series 2024 7.0 ) Mark Scored : 86.90

A. Only one
B. Only two
C. All three
D. None

Your Answer : C
Correct Answer : A

Answer Justification :

Statement 1 is correct: The Namdapha Flying Squirrel, missing for four decades, has
resurfaced in Arunachal Pradesh. Last documented in 1981, this nocturnal flying squirrel is
indigenous to the Changlang district and endemic to Arunachal Pradesh, specifically the
Namdapha National Park.

7
Statement 2 is not correct: The Namdapha Flying Squirrel is often confused with the2Red
77
3
Giant Flying Squirrel, another species found in the region. 06
9 56
Distinguishing between the two presents a considerable challenge due to their similar -
m
appearances. .co il
m a
4 ;g
6
The Namdapha Flying Squirrel has reddish grizzled &#
5 fur with white markings. Its crown displays a
a 9
s tav or fold of skin between the forelimbs and
pale grey hue, while its patagium (a membrane
hindlimbs) exhibits an orangish color, a
and its underparts are white.
sr iv
ti
Statement 3 is correct: The
h ru squirrel is among the ‘25 most wanted lost species’ identified by
s
the Global Wildlife Conservation’s Search for Lost Species initiative.
v a-
Additionally, itsista
i v a asprotected under Schedule I of the Wildlife Protection (Amendment) Act 2022

Sr
and is classified critically endangered on the IUCN Red List.
i
r ut
Sh
https://prsindia.org/files/bills_acts/acts_parliament/2022/The%20Wild%20Life%20(Protection)%2
0Amendment%20Act,%202022.pdf

64. Consider the following statements:


The Leif Erikson Lunar Prize-
1. Is awarded annually by the Exploration Museum for achievements in exploration.
2. Has been awarded to the Indian Space Research Organisation for Chandrayaan 3.

Which of the statements given above is/are correct?


A. 1 only
B. 2 only
C. Both 1 and 2
D. Neither 1 nor 2

instacourses.insightsonindia.com 51
© Insights Active Learning | All rights reserved - 18107. You may not reproduce, distribute or exploit the contents in any
form without written permission by copyright owner. Copyright infringers may face civil and criminal liability
Total Marks : 200.00
Prelims Test 23 - GS Test 16
( Insta Prelims Test Series 2024 7.0 ) Mark Scored : 86.90

Your Answer :
Correct Answer : C

Answer Justification :

Option (c) is correct: For its ‘s indomitable spirit in advancing lunar exploration & contributing
to understanding celestial mysteries and in celebration the first soft landing of Chandrayaan 3,
Indian Space Research Organisation (ISRO) has been awarded Leif Erikson Lunar Prize. This is
the first time that ISRO has received this award.

This award is known as the Exploration Awards, and is awarded annually by the Exploration
Museum for achievements in exploration and for work in the field of exploration history.

Awarded annually in Iceland they are in three categories including:

the Leif Erikson Award to an explorer for a lifetime achievement in exploration; 77


7
6 32
Leif Erikson Young Explorer Award to an explorer under the age of 35 for achievements in
6 0
exploration; 5
and the Leif Erikson Exploration History Award – this is presented to -a 9
person or an
organization that has worked to promote and preserve exploration
.c omhistory, or to educate
about exploration, science and environment issues. ai l
4 ;gm
& #6
65. With reference to Taraknath Das, consider the
9 5 following statements:
a
tavnewspaper, ‘Free Hindusthan’.
1. He was associated with the Ghadar Movement and its anti-colonial activities.
s
va society called Anushilan Samiti.
2. He started the publication of the anti-British
r i
tis
3. He was a founding member of the Secret
u
shr given above is/are correct?
a-
How many of the statements
A. Only one v
B. Only two a sta
iv
rthree
S
C. All
i
r uD.t None
Sh
Your Answer : B
Correct Answer : B

Answer Justification :

Taraknath Das was an Indian revolutionary with a global outlook.

Statement 1 is correct: In 1913, Das came in contact with Lala Har Dayal and got associated
with the Ghadar Movement and its anti-colonial activities.

In 1917, he was implicated in the Indo-German conspiracy case for which he was imprisoned in
Kansas for two years.

Statement 2 is correct: Following the Bellingham riots of September 1907 against South

instacourses.insightsonindia.com 52
© Insights Active Learning | All rights reserved - 18107. You may not reproduce, distribute or exploit the contents in any
form without written permission by copyright owner. Copyright infringers may face civil and criminal liability
Total Marks : 200.00
Prelims Test 23 - GS Test 16
( Insta Prelims Test Series 2024 7.0 ) Mark Scored : 86.90

Asian immigrants, he started the publication of an anti-British newspaper, ‘Free Hindusthan’ to


champion the cause of these immigrants.

Statement 3 is not correct: At a young age Das was attracted by the revolutionary cause of
the Anushilan Samiti, a secret society, and became its member.

66. Consider the following statements regarding the Kudankulam Nuclear Power Plant:
1. It is India's largest nuclear power plant and is being built in Kerala.
2. It is being constructed with the technical assistance of Russia.

Which of the statements given above is/are correct?


A. 1 only
B. 2 only 7
C. Both 1 and 2 277
6 3
60
D. Neither 1 nor 2
9 5
Your Answer : C -
m
Correct Answer : B
il.co
m a
Answer Justification : 4 ;g
6
&#
Option (b) is correct: The Kudankulam nuclear
a95power plant, India's largest, is being built in
v
a Russia. The construction began in March 2002.
Tamil Nadu with the technical assistance of
ast
v
ri unit of the Kudankulam NPP has been steadily operating at
s
Since February 2016, the first power
i
ut The plant is expected to start operating at full capacity in
its design capacity of 1,000rMW.
h
2027.
a -s
stav
a
Sriv
i
ut the following statements:
67. Consider
r
ShThe RBI Retail Direct Scheme facilitates investment in Government Securities by individual
1.
investors.
2. Promoting retail participation in the G-sec market is beneficial to both the issuer and the
investor.
3. The Regional Rural Banks are not eligible to invest in the Government securities (G-sec) market
in India.

How many of the statements given above are correct?


A. Only one
B. Only two
C. All three
D. None

Your Answer : B
Correct Answer : B

instacourses.insightsonindia.com 53
© Insights Active Learning | All rights reserved - 18107. You may not reproduce, distribute or exploit the contents in any
form without written permission by copyright owner. Copyright infringers may face civil and criminal liability
Total Marks : 200.00
Prelims Test 23 - GS Test 16
( Insta Prelims Test Series 2024 7.0 ) Mark Scored : 86.90

Answer Justification :

Statement 1 is correct: Retail Direct Scheme is a one-stop solution to facilitate investment in


Government Securities by individual investors.

Under this scheme individual retail investors can open a Gilt Securities Account –Retail
Direct Gilt (RDG) account with RBI.
Statement 2 is correct: Promoting retail participation in the G-sec market is beneficial
to both the issuer and the investor.
From the issuer‘s perspective, a diversified investor base for government bonds ensures
stable demand for G-sec.
Statement 3 is not correct: The investors in Government securities (G-sec) market in
India are mostly institutions like commercial banks, co-operative banks, regional rural
banks, provident funds, pension funds, mutual funds and non-banking finance companies.
Retail participation, i.e., participation by individuals in the G-sec market has been very7
limited till now. 2 77
3
G-Secs issued by the Central government excluding Treasury Bills will now be
6 06eligible for
lending/borrowing under a Government Security Lending (GSL) transaction. 95 -
m
l.co
68. With reference to Ammonia, consider the following statements:
m ai
1. It occurs naturally and is also produced by human activity.
4 ;g
2. It can be dissolved in water in the gaseous form.
& #6
5
3. It can be produced by the bacteria found in the9intestines.
ta va
How many of the statements given above
i vas are not correct?
A. Only one tisr
u
B. Only two
shr
C. All three
v a-
D. None
a sta
S riv : B
i
Your Answer
r ut Answer : D
Correct
Sh
Answer Justification :

Option (d) is correct: Ammonia occurs naturally and is produced by human activity.

It is an important source of nitrogen which is needed by plants and animals.


Ammonia gas can be dissolved in water.
This kind of ammonia is called liquid ammonia or aqueous ammonia.
Bacteria found in the intestines can produce ammonia.
Ammonia is a colorless gas with a very distinct odor. This odor is familiar to many people
because ammonia is used in smelling salts, many household and industrial cleaners, and
window-cleaning products.

69. Consider the following:

instacourses.insightsonindia.com 54
© Insights Active Learning | All rights reserved - 18107. You may not reproduce, distribute or exploit the contents in any
form without written permission by copyright owner. Copyright infringers may face civil and criminal liability
Total Marks : 200.00
Prelims Test 23 - GS Test 16
( Insta Prelims Test Series 2024 7.0 ) Mark Scored : 86.90

1. Russia
2. China
3. Kuwait
4. Oman
5. Saudi Arabia

How many of the countries mentioned above are a part of the Organization of the Petroleum
Exporting Countries (OPEC+)?
A. Only two
B. Only three
C. Only four
D. All five

Your Answer : B
7
Correct Answer : C
277
6 3
Answer Justification :
560
- 9
Option (c) is correct: OPEC, which stands for the Organization of them Petroleum Exporting
c o
il. Arabia, and Venezuela.
Countries, was formed in Baghdad in 1960 by Iraq, Iran, Kuwait, Saudi
a
g m
OPEC currently has 13 members, with the five founders
6 4; having since been joined by
&# Libya, Nigeria and the United Arab
Algeria, Angola, Congo, Equatorial Guinea, Gabon,
5
a9 capital Vienna.
Emirates. Its headquarters is in the Austrian
v
sta
va
i sri
In 2016, OPEC formedran utalliance with other oil-producing nations to create OPEC+. The
h
s OPEC+ include Russia, Kazakhstan, Azerbaijan, Mexico, and Oman.
10 countries now- in
a
tavcreate OPEC+ was a response to falling crude oil prices partly caused by a
The move to
s
r i a
huge vincrease in US shale oil production since 2011.
iS
r ut
h
70.SRecently numerous SAMARTH Curated Programs were launched by NITI Aayog in
collaboration with-

A. Bharatmala Pariyojana
B. Karmayogi Bharat
C. Ek Bharat Shreshtha Bharat
D. Samagra Shiksha

Your Answer : D
Correct Answer : B

Answer Justification :

Option (b) is correct: Karmayogi Bharat, in association with NITI Aayog, has launched 6
SAMARTH Curated Training Programs on the iGOT Karmayogi Platform for State Government

instacourses.insightsonindia.com 55
© Insights Active Learning | All rights reserved - 18107. You may not reproduce, distribute or exploit the contents in any
form without written permission by copyright owner. Copyright infringers may face civil and criminal liability
Total Marks : 200.00
Prelims Test 23 - GS Test 16
( Insta Prelims Test Series 2024 7.0 ) Mark Scored : 86.90

officials.

The programs are- 1) SAMARTH Blocks, 2) SAMARTH Zila, 3) SAMARTH Rajya, 4) SAMARTH
Rajya Sachiva, 5) SAMARTH Policy, and 6) SAMARTH Procurement.
These programs aim to elevate the skills and competencies of government officials,
ensuring they are well-equipped to meet the evolving needs of governance.

71. With reference to the Pitt’s India Act of 1784, consider the following statements:
1. It initiated the process of decentralization by restoring the legislative powers to the Bombay and
Madras Presidencies.
2. It introduced local representation in the Indian Legislative Council and ended the activities of
the East India Company as a commercial body.

Which of the statements given above is/are correct? 7


A. 1 only 277
6 3
60
B. 2 only
C. Both 1 and 2 9 5
-
D. Neither 1 nor 2 m
il.co
Your Answer : D m a
Correct Answer : D 4 ;g
6
5 &#
Answer Justification :
va9
s ta
r va
In a bid to rectify the defects of the iRegulating Act of 1773, the British Parliament passed the
Amending Act of 1781, also known ti s as the Act of Settlement.
u
shr
The next important act- was the Pitt’s India Act of 1784.
t a va
i v as
It distinguished between the commercial and political functions of the Company.
r
ItSallowed the Court of Directors to manage the commercial affairs but created a new
t ibody
h ru called Board of Control to manage the political affairs.
S Thus, it established a system of double government.
It empowered the Board of Control to supervise and direct all operations of the civil and
military government or revenues of the British possessions in India.

Statement 1 is not correct: Indian Councils Act of 1861 initiated the process of
decentralisation by restoring the legislative powers to the Bombay and Madras Presidencies.

It thus reversed the centralising tendency that started from the Regulating Act of 1773
and reached its climax under the Charter Act of 1833.

Statement 2 is not correct: The Charter Act of 1853 introduced, for the first time, local
representation in the Indian (Central) Legislative Council.

Of the six new legislative members of the governor-general’s council, four members were
appointed by the local (provincial) governments of Madras, Bombay, Bengal and Agra.

instacourses.insightsonindia.com 56
© Insights Active Learning | All rights reserved - 18107. You may not reproduce, distribute or exploit the contents in any
form without written permission by copyright owner. Copyright infringers may face civil and criminal liability
Total Marks : 200.00
Prelims Test 23 - GS Test 16
( Insta Prelims Test Series 2024 7.0 ) Mark Scored : 86.90

72. Consider the following statements regarding the Government of India Act of 1935:
1. It provided for the establishment of an All-India Federation consisting of princely states as units.
2. It abolished diarchy in the provinces and introduced the system of ‘provincial autonomy’ in its
place.
3. It extended the principle of communal representation for the depressed classes by providing
separate electorate.

How many of the statements given above is/are correct?


A. Only one
B. Only two
C. All three
D. None

Your Answer : B 7
Correct Answer : C 277
6 3
560
Answer Justification :
- 9
m
Option (c) is correct: The Government of India Act of 1935 marked
i l .coa second milestone
towards a completely responsible government in India. It wasm aalengthy and detailed document
having 321 Sections and 10 Schedules. 4;g
6
5 &#Federation consisting of provinces and
a9
It provided for the establishment of an All-India
princely states as units. a v
ast the Centre and units in terms of three lists—Federal
v
The Act divided the powers between
ri Provincial List (for provinces, with 54 items) and the
i s
List (for Centre, with 59 items),
t
Concurrent List (for h ru with 36 items).
both,
- s in the provinces and introduced ‘provincial autonomy’ in its place.
It abolished dyarchy
a
tav were allowed to act as autonomous units of administration in their defined
The provinces
s
a
riv extended the principle of communal representation by providing separate
spheres.
iItSfurther
r ut electorates for depressed classes (scheduled castes), women and labour (workers).
Sh
73. Consider the following statements:
#18368
1. The Constituent Assembly was constituted under the scheme formulated by the Cabinet Mission
Plan.
2. The members of the Constituent Assembly were indirectly elected by the members of the
provincial assemblies.
3. The Constituent Assembly originally declared its solemn resolve to proclaim India as a Socialist
Sovereign Republic.

How many of the statements given above are correct?


A. Only one
B. Only two
C. All three

instacourses.insightsonindia.com 57
© Insights Active Learning | All rights reserved - 18107. You may not reproduce, distribute or exploit the contents in any
form without written permission by copyright owner. Copyright infringers may face civil and criminal liability
Total Marks : 200.00
Prelims Test 23 - GS Test 16
( Insta Prelims Test Series 2024 7.0 ) Mark Scored : 86.90

D. None

Your Answer : B
Correct Answer : B

Answer Justification :

Statement 1 is correct: The Constituent Assembly was constituted in November 1946 under
the scheme formulated by the Cabinet Mission Plan.

Statement 2 is correct: The Constituent Assembly was partly elected and partly nominated
body. Moreover, the members were to be indirectly elected by the members of the provincial
assemblies, who themselves were elected on a limited franchise 4

On December 13, 1946, Jawaharlal Nehru moved the historic ‘Objectives Resolution’ in the
7
77
Assembly. It laid down the fundamentals and philosophy of the constitutional structure. It read:
2
3
06
Statement 3 is not correct: “This Constituent Assembly declares its firm and
9 56solemn resolve
- not adopted by the
to proclaim India as an Independent Sovereign Republic (Socialist term was
m
Constituent Assembly). co il.
m a
4 ;g
# 6
74. With reference to Indian Constitution, consider & the following statements:
1. The Federal Scheme and emergency powersva 95 been drawn from the Government of India
have
Act, 1935.
a sta
r
2. The political part including principle v Cabinet Government has been drawn from the British
iof
i s
Constitution.
r ut
- sh
3. The Indian Constitution contains federal features that include the flexibility of Constitution and
a
tav
integrated judiciary.
s
How many vathe statements given above is/are not correct?
riof
i S
r uA.t Only one
Sh B. Only two
C. All three
D. None

Your Answer : D
Correct Answer : A

Answer Justification :

Statement 1 is correct: The most profound influence and material source of the Indian
Constitution is the Government of India Act, 1935.

The Federal Scheme, Judiciary, Governors, emergency powers, the Public Service Commissions
and most of the administrative details are drawn from this Act.

instacourses.insightsonindia.com 58
© Insights Active Learning | All rights reserved - 18107. You may not reproduce, distribute or exploit the contents in any
form without written permission by copyright owner. Copyright infringers may face civil and criminal liability
Total Marks : 200.00
Prelims Test 23 - GS Test 16
( Insta Prelims Test Series 2024 7.0 ) Mark Scored : 86.90

Statement 2 is correct: The political part of the Constitution (the principle of Cabinet
Government and the relations between the executive and the legislature) have been largely
drawn from the British Constitution

Statement 3 is not correct: The Indian Constitution also contains a large number of unitary
or non-federal features, viz., a strong Centre, single Constitution, single citizenship, flexibility of
Constitution, integrated judiciary, appointment of state governor by the Centre, all-India
services, emergency provisions, and so on.

75. Consider the following statements:


1. Unlike the presidential system, the parliamentary system is based on the doctrine of separation
of powers between the legislative and executive organs.
2. Even though the Indian Parliamentary System is largely based on the British pattern, it is not
7a
7
sovereign body like the British Parliament. 27
63
Which of the statements given above is/are correct? 560
- 9
A. 1 only
m
B. 2 only
il.co
C. Both 1 and 2
m a
D. Neither 1 nor 2 4 ;g
6
5 &#
Your Answer : B
va9
Correct Answer : B ta
i vas
i sr
ut
Answer Justification :
h r
-
The Constitution of Indiashas opted for the British parliamentary System of Government rather
v a
ta Presidential System of Government.
than the American
s
a
S
Statementriv1 is not correct: The parliamentary system is based on the principle of
i
r ut
cooperation and co-ordination between the legislative and executive organs while the
h
S presidential system is based on the doctrine of separation of powers between the two organs.
Statement 2 is correct: Even though the Indian Parliamentary System is largely based on the
British pattern, there are some fundamental differences between the two. For example, the
Indian Parliament is not a sovereign body like the British Parliament.

Further, the Indian State has an elected head (republic) while the British State has a hereditary
head (monarchy).

76. Consider the following statements regarding the Indian Constitution:


1. The original constitution did not provide for the fundamental duties of the citizens.
2. Unlike the Directive Principles, the fundamental duties are justiciable in nature.
3. The 86th Constitutional Amendment Act, 2002 added Socialist Directive Principles to the

instacourses.insightsonindia.com 59
© Insights Active Learning | All rights reserved - 18107. You may not reproduce, distribute or exploit the contents in any
form without written permission by copyright owner. Copyright infringers may face civil and criminal liability
Total Marks : 200.00
Prelims Test 23 - GS Test 16
( Insta Prelims Test Series 2024 7.0 ) Mark Scored : 86.90

Constitution.

How many of the statements given above are correct?


A. Only one
B. Only two
C. All three
D. None

Your Answer : A
Correct Answer : A

Answer Justification :

Statement 1 is correct: The original constitution did not provide for the fundamental duties of
the citizens. These were added during the operation of internal emergency (1975–77) by the7
42nd Constitutional Amendment Act of 1976 on the recommendation of the Swaran Singh 277
3
Committee. 06
9 56
Statement 2 is not correct: The fundamental duties serve as a reminder -
to citizens that
c om they owe to their
while enjoying their rights, they have also to be quite conscious ofl.duties
country, their society and to their fellow-citizens. However, like
i Directive Principles, the
athe
m
duties are also non-justiciable in nature. 4;g
& #6
a95
Statement 3 is not correct: The 86th Constitutional Amendment Act of 2002 added one more
v
sta
fundamental duty.
iva
i sr
r ut
77. The provision “Federation
- sh with a strong Centre” has been borrowed from
t a va
A. British a s
Constitution
i v
i Sr
B. Canadian Constitution
t
ruC. Japanese Constitution
Sh D. Australian Constitution
Your Answer : B
Correct Answer : B

Answer Justification :
Sources Features Borrowed
British Constitution Parliamentary government, Rule of Law, legislative procedure,
single citizenship, cabinet system, prerogative writs,
parliamentary privileges and bicameralism.
Canadian Constitution Federation with a strong Centre, vesting of residuary
powers in the Centre, appointment of state governors
by the Centre, and advisory jurisdiction of the Supreme
Court.

instacourses.insightsonindia.com 60
© Insights Active Learning | All rights reserved - 18107. You may not reproduce, distribute or exploit the contents in any
form without written permission by copyright owner. Copyright infringers may face civil and criminal liability
Total Marks : 200.00
Prelims Test 23 - GS Test 16
( Insta Prelims Test Series 2024 7.0 ) Mark Scored : 86.90

Australian Constitution Concurrent List, freedom of trade, commerce and inter-course,


and joint sitting of the two Houses of Parliament.
Japanese Constitution Procedure established by Law.

Weimar Constitution of Suspension of Fundamental Rights during Emergency.


Germany

78. Consider the following statements:


1. The 73rd Constitutional Amendment Act added a third-tier of government which is not found in
any other Constitution of the world.
2. The Tenth Schedule in the Indian Constitution was added by the 52nd Amendment Act of 1985
also known as Anti-defection Law.

7
Which of the statements given above is/are correct?
277
A. 1 only
6 3
B. 2 only
560
C. Both 1 and 2 - 9
m
.co
D. Neither 1 nor 2
ail
Your Answer : C
;g m
Correct Answer : C 6 4
5 &#
Answer Justification : va9
ta
as
ivthe Indian Constitution, like any other federal constitution,
Statement 1 is correct: Originally,
sr
ti
provided for a dual polity anducontained provisions with regard to the organisation and powers
h r
s
of the Centre and the states.
v a-
ta 74th Constitutional Amendment Acts (1992) have added a third-tier of
Later, the 73rdsand
a
riv (i.e., local) which is not found in any other Constitution of the world.
government
S
i
r ut
Statement 2 is correct: The Provisions of the Tenth Schedule of Indian Constitution relate to
Shdisqualification of the members of Parliament and State Legislatures on the ground of defection.
This schedule was added by the 52nd Amendment Act of 1985, also known as Anti-defection
Law.

79. Which one of the following terms was not added to the Indian Preamble by the 42nd
Constitutional Amendment Act (1976)?

A. Integrity
B. Socialist
C. Secular
D. Fraternity

instacourses.insightsonindia.com 61
© Insights Active Learning | All rights reserved - 18107. You may not reproduce, distribute or exploit the contents in any
form without written permission by copyright owner. Copyright infringers may face civil and criminal liability
Total Marks : 200.00
Prelims Test 23 - GS Test 16
( Insta Prelims Test Series 2024 7.0 ) Mark Scored : 86.90

Your Answer : D
Correct Answer : D

Answer Justification :

Option (d) is correct: The Preamble to the Indian Constitution is based on the ‘Objectives
Resolution’, drafted and moved by Pandit Nehru, and adopted by the Constituent Assembly.

It has been amended by the 42nd Constitutional Amendment Act (1976), which added
three new words—socialist, secular and integrity.
The Preamble to the Indian Constitution is based on the ‘Objectives Resolution’, drafted
and moved by Pandit Nehru, and adopted by the Constituent.
The American Constitution was the first to begin with a Preamble. Many countries,
including India, followed this practice.
The term ‘preamble’ refers to the introduction or preface to the Constitution. 7
277
3
06
80. Consider the following statements:
9 56
-
1. The Indian Preamble is a source of power to the legislature but not a prohibition upon the
om
il.ctaken from the French
powers of the legislature.
a
2. The ideals of liberty and equality in the Indian Preamble have been
Revolution (1789–1799). 4 ;gm
& #6
3. The ideal of social and political justice in the Indian Preamble has been taken from the Russian
Revolution (1917). 9 5
ta va
How many of the statements given above
i vas are correct?
A. Only one tisr
u
B. Only two
shr
C. All three
v a-
D. None
a sta
S riv : B
i
Your Answer
r ut Answer : B
Correct
Sh
Answer Justification :

Option (b) is correct: The Preamble embodies the basic philosophy and fundamental values
political, moral and religious on which the Constitution is based.

The Preamble is neither a source of power to legislature nor a prohibition upon the powers
of legislature.
It is non-justiciable, that is, its provisions are not enforceable in courts of law.
The ideals of liberty, equality and fraternity in our Preamble have been taken from the
French Revolution (1789–1799).
The ideal of justice—social, economic and political—has been taken from the Russian
Revolution (1917)

instacourses.insightsonindia.com 62
© Insights Active Learning | All rights reserved - 18107. You may not reproduce, distribute or exploit the contents in any
form without written permission by copyright owner. Copyright infringers may face civil and criminal liability
Total Marks : 200.00
Prelims Test 23 - GS Test 16
( Insta Prelims Test Series 2024 7.0 ) Mark Scored : 86.90

81. Consider the following statements:


1. Being a sovereign state, India can acquire foreign territories through cession and occupation
and subjugation.
2. The Indian Constitution has restricted the Parliament to alter the areas of existing states
without their consent.

Which of the statements given above is/are correct?


A. 1 only
B. 2 only
C. Both 1 and 2
D. Neither 1 nor 2

Your Answer : A
Correct Answer : A 7
277
6 3
Answer Justification :
560
9
- territories
Statement 1 is correct: Being a sovereign state, India can acquire foreign
m
according to the modes recognized by international law, i.e., cession
i l .co(following treaty,
a by a recognised ruler),
purchase, gift, lease or plebiscite), occupation (hitherto unoccupied
m
conquest or subjugation. 4;g
& #6
Statement 2 is not correct: It is thus clear that
a 95 the Constitution authorises the Parliament to
v
sta
form new states or alter the areas, boundaries or names of the existing states without their
consent.
iva
i sr
r
The territorial integrity uort continued existence of any state is not guaranteed by the
- sh India is rightly described as ‘an indestructible union of
Constitution. Therefore,
destructibleav a
states’.
t
i v as government can destroy the states whereas the state governments cannot
The Union

i Sr the Union.
destroy
t
h ru
S
82. The concept of ‘equality before law’ is an element of the concept of ‘Rule of Law’ and
this concept best includes-

A. Absence of arbitrary power


B. Equal subjection of citizens
C. Primacy of the rights of individuals
D. All of the above

Your Answer : D
Correct Answer : D

Answer Justification :

Option (d) is correct: Rule of Law The concept of ‘equality before law’ is an element of the

instacourses.insightsonindia.com 63
© Insights Active Learning | All rights reserved - 18107. You may not reproduce, distribute or exploit the contents in any
form without written permission by copyright owner. Copyright infringers may face civil and criminal liability
Total Marks : 200.00
Prelims Test 23 - GS Test 16
( Insta Prelims Test Series 2024 7.0 ) Mark Scored : 86.90

concept of ‘Rule of Law’. The concept has the following three elements or aspects:

Absence of arbitrary power, that is, no man can be punished except for a breach of law.
Equality before the law, that is, equal subjection of all citizens (rich or poor, high or low,
official or non-official) to the ordinary law of the land administered by the ordinary law
courts.
The primacy of the rights of the individual, that is, the constitution is the result of the
rights of the individual as defined and enforced by the courts.

83. Consider the following:


The state is permitted to make any special provision for women and children and is
also permitted to make special provisions for the scheduled castes.
Which one of the following Articles of the Indian Constitution is associated with the
exception given above?
7
277
A. Article 14 6 3
B. Article 15 560
C. Article 16 - 9
m
D. Article 17
il.co
m a
Your Answer : B
4 ;g
6
&#
Correct Answer : B
5
va9
Answer Justification :
ta
i vas
Option (b) is correct: Article 15srprovides that the State shall not discriminate against any
i
r ut race, caste, sex or place of birth.
citizen on grounds only of religion,
- sh
t a va
There are three exceptions to this general rule of non-discrimination:
s
va is permitted to make any special provision for women and children. For
Theristate
S
iexample,
t
u children.
reservation of seats for women in local bodies or provision of free education for
r
Sh The state is permitted to make any special provision for the advancement of any socially
and educationally backward classes of citizens or for the scheduled castes and scheduled
tribes.
The state is empowered to make any special provision for the advancement of any
socially and educationally backward classes of citizens or for the scheduled castes or the
scheduled tribes regarding their admission to educational institutions.

84. Consider the following:


1. Right to elementary education
2. Abolition of untouchability
3. Protection in respect of conviction for offences
4. Prohibition of employment of children in factories

instacourses.insightsonindia.com 64
© Insights Active Learning | All rights reserved - 18107. You may not reproduce, distribute or exploit the contents in any
form without written permission by copyright owner. Copyright infringers may face civil and criminal liability
Total Marks : 200.00
Prelims Test 23 - GS Test 16
( Insta Prelims Test Series 2024 7.0 ) Mark Scored : 86.90

How many of the Fundamental Rights given above do not fall under the Right against
Exploitation?
A. Only one
B. Only two
C. Only three
D. All four

Your Answer : C
Correct Answer : C

Answer Justification :

Option (c) is correct: Right Against Exploitation covers Prohibition of Traffic in Human Beings
and Forced Labour and Prohibition of Employment of Children in Factories, etc.
77
Article 23 prohibits traffic in human beings and other similar forms of forced labour.
3 27
Any contravention of this provision shall be an offence punishable in accordance
6 06 with law.
This right is available to both citizens and non-citizens.
- 95
It protects the individual not only against the State but also againstm private persons.
c o
Article 24 prohibits the employment of children below the age
a i l. 14 years in any factory,
of
mine or other hazardous activities like construction work m or railway.
4 ;
But it does not prohibit their employment in any harmlessg or innocent work.
& #6
5
a9
85. Consider the following statements:
s tav
r va
1. Article 29 of the Indian Constitutioniprovides that the citizens residing in any part of India shall
s
have the right to conserve theirtidistinct language, script or culture.
u
2. The political speeches made
shr for the conservation of any language amount to corrupt practice
under the Representation- of the People Act, 1951.
t a va
s
astatements
Which of the
r i v given above is/are correct?
S
A. i1 only
u t
h r B. 2 only
S C. Both 1 and 2
D. Neither 1 nor 2

Your Answer : A
Correct Answer : A

Answer Justification :

Statement 1 is correct: Article 29 provides that any section of the citizens residing in any
part of India having a distinct language, script or culture of its own, shall have the right to
conserve the same.

Further, no citizen shall be denied admission into any educational institution maintained
by the State or receiving aid out of State funds on grounds only of religion, race, caste, or
language.

instacourses.insightsonindia.com 65
© Insights Active Learning | All rights reserved - 18107. You may not reproduce, distribute or exploit the contents in any
form without written permission by copyright owner. Copyright infringers may face civil and criminal liability
Total Marks : 200.00
Prelims Test 23 - GS Test 16
( Insta Prelims Test Series 2024 7.0 ) Mark Scored : 86.90

Statement 2 is not correct: The Supreme Court also held that the right to conserve the
language includes the right to agitate for the protection of the language.

Hence, the political speeches or promises made for the conservation of the language of a
section of the citizens does not amount to corrupt practice under the Representation of
the People Act, 1951.

86. Consider the following statements:


1. The concept of writs is borrowed from the British law where they are known as prerogative
writs.
2. The Supreme Court can issue the writs only for the enforcement of Fundamental Rights.
3. The High courts unlike the Supreme Court cannot refuse to exercise their writ jurisdiction.

How many of the statements given above are correct? 7


A. Only one 277
6 3
B. Only two
560
C. All three
- 9
D. None m
il.co
Your Answer : A m a
Correct Answer : B 4 ;g
6
5 &#
Answer Justification :
va9
s ta
Statement 1 is correct: The writsiare
r va borrowed from English law where they are known as
s in England as they were issued in the exercise of the
‘prerogative writs’. They are soticalled
u
prerogative of the King whohrwas, and is still, described as the ‘fountain of justice’.
a -s
Statement 2 ista v
correct: The Supreme Court can issue writs only for the enforcement of
s
arights whereas a high court can issue writs not only for the enforcement of
riv Rights but also for any other purpose.
fundamental
S
Fundamental
i
r ut
ShA remedy under Article 32 is in itself a Fundamental Right and hence, the Supreme Court may
not refuse to exercise its writ jurisdiction.

Statement 3 is not correct: On the other hand, a remedy under Article 226 is discretionary
and hence, a high court may refuse to exercise its writ jurisdiction.

87. With reference to Indian Constitution, consider the following statements:


1. The power to make laws under Article 33 is conferred only on Parliament and not on state
legislatures.
2. A parliamentary law enacted under Article 33 can exclude the court martials from the writ
jurisdiction of the Supreme Court.
3. Article 33 provides for the restrictions on fundamental rights while martial law is in force within
the territory of India.

instacourses.insightsonindia.com 66
© Insights Active Learning | All rights reserved - 18107. You may not reproduce, distribute or exploit the contents in any
form without written permission by copyright owner. Copyright infringers may face civil and criminal liability
Total Marks : 200.00
Prelims Test 23 - GS Test 16
( Insta Prelims Test Series 2024 7.0 ) Mark Scored : 86.90

How many of the statements given above are not correct?


A. Only one
B. Only two
C. All three
D. None

Your Answer : D
Correct Answer : A

Answer Justification :

Article 33 empowers the Parliament to restrict or abrogate the fundamental rights of the
members of armed forces, para-military forces, police forces, intelligence agencies and
analogous forces.
77
Statement 1 is correct: The power to make laws under Article 33 is conferred only on
3 27
Parliament and not on state legislatures. Any such law made by Parliament cannot
6 06be
9 5
challenged in any court on the ground of contravention of any of the fundamental rights.
-
m
Statement 2 is correct: A parliamentary law enacted under Article
i l .co33 can also exclude the
a the writ jurisdiction of the
court martials (tribunals established under the military law) from
m
g
4; of Fundamental Rights is
Supreme Court and the high courts, so far as the enforcement
6
concerned.
5 &#
va9 for the restrictions on fundamental rights
a
Statement 3 is not correct: Article 34 provides
t the territory of India.
swithin
while martial law is in force in any area
va
i sri
It empowers the Parliamentrto utindemnify any government servant or any other person for any
h
a -s
act done by him in connection with the maintenance or restoration of order in any area where
martial law was in v
force.
a sta
S riv
ti
u44th
r
Sh
88. The Amendment Act of 1978 of the Indian Constitution-

A. Abolished the Right to property as a Fundamental Right.


B. Transferred education to Concurrent List from State List.
C. Made a National Emergency immune from the judicial review.
D. Made the President bound by the advice of the council of ministers.

Your Answer : A
Correct Answer : A

Answer Justification :

Option (a) is correct: Originally, the right to property was one of the seven fundamental
rights under Part III of the Constitution.

instacourses.insightsonindia.com 67
© Insights Active Learning | All rights reserved - 18107. You may not reproduce, distribute or exploit the contents in any
form without written permission by copyright owner. Copyright infringers may face civil and criminal liability
Total Marks : 200.00
Prelims Test 23 - GS Test 16
( Insta Prelims Test Series 2024 7.0 ) Mark Scored : 86.90

It was dealt by Article 19(1)(f) and Article 31. Article 19(1)(f) guaranteed to every citizen the
right to acquire, hold and dispose of property.

Therefore, the 44th Amendment Act of 1978 abolished the right to property as a Fundamental
Right by repealing Article 19(1)(f) and Article 31 from Part III.

Instead, the Act inserted a new Article 300A in Part XII under the heading ‘Right to Property’. It
provides that no person shall be deprived of his property except by authority of law.

89. Consider the following:


1. No tax shall be levied or collected except by authority of law.
2. No person shall be deprived of his property save by authority of law.
3. The elections to the Lok Sabha shall be held based on adult suffrage.
7
277
How many of the Rights given above are legal or Non-Fundamental Rights? 6 3
A. Only one 560
B. Only two - 9
m
C. All three
i l .co
D. None a m
4 ;g
6
&#
Your Answer : C
5
a9
Correct Answer : C
ta v
Answer Justification :
i vas
t isr
ru the Fundamental Rights included in Part III, there are certain
Option (c) is correct: Besides
h
other rights contained -insother parts of the Constitution.

t a va
These rights a s known as constitutional rights or legal rights or non-fundamental rights.
are
i v
Sr
Theytiare:
ru
Sh No tax shall be levied or collected except by authority of law (Article 265 in Part XII).
No person shall be deprived of his property save by authority of law (Article 300-A in Part
XII).
Trade, commerce and intercourse throughout the territory of India shall be free (Article
301 in Part XIII).
The elections to the Lok Sabha and the State Legislative Assembly shall be on the basis of
adult suffrage (Article 326 in Part XV).

90. Consider the following statements:


1. According to Article 36 of the Indian Constitution, the term ‘State’ in Part IV has the same
meaning as in Part III dealing with Fundamental Rights.
2. The Directive Principles of State Policy are non-justiciable in nature but help the courts in
determining the constitutional validity of a law.

instacourses.insightsonindia.com 68
© Insights Active Learning | All rights reserved - 18107. You may not reproduce, distribute or exploit the contents in any
form without written permission by copyright owner. Copyright infringers may face civil and criminal liability
Total Marks : 200.00
Prelims Test 23 - GS Test 16
( Insta Prelims Test Series 2024 7.0 ) Mark Scored : 86.90

Which of the statements given above is/are correct?


A. 1 only
B. 2 only
C. Both 1 and 2
D. Neither 1 nor 2

Your Answer : C
Correct Answer : C

Answer Justification :

Option (c) is correct: According to Article 36, the term ‘State’ in Part IV has the same
meaning as in Part III dealing with Fundamental Rights. Therefore, it includes the legislative and
executive organs of the central and state governments, all local authorities and all other public
authorities in the country. 7
277
3
06
The Directive Principles constitute a very comprehensive economic, social and political
6
programme for a modern democratic State. They aim at realising the high ideals
- 95 of justice,
liberty, equality and fraternity as outlined in the Preamble to the Constitution.
m
i l .co
a courts in examining and
The Directive Principles, though non-justiciable in nature, help the
m
determining the constitutional validity of a law. 4; g
6
5 &#
va9
st a
va
i sri
t
91. Which one of the followinguArticles of the Indian Constitution does not uphold the
ideology of liberalism? shr
v a-
A. Article a sta
43
riv 44
B. Article
S
i
r uC.t Article 45
Sh D. Article 48
Your Answer : A
Correct Answer : A

Answer Justification :

Option (a) is correct: Liberal–Intellectual Principles The principles included in this category
represent the ideology of liberalism.

They direct the state:

To secure for all citizens a uniform civil code throughout the country (Article 44).
To provide early childhood care and education for all children until they complete the age
of six years 9 (Article 45).

instacourses.insightsonindia.com 69
© Insights Active Learning | All rights reserved - 18107. You may not reproduce, distribute or exploit the contents in any
form without written permission by copyright owner. Copyright infringers may face civil and criminal liability
Total Marks : 200.00
Prelims Test 23 - GS Test 16
( Insta Prelims Test Series 2024 7.0 ) Mark Scored : 86.90

To organise agriculture and animal husbandry on modern and scientific lines (Article 48).
To protect and improve the environment and to safeguard forests and wild life 10 (Article
48 A).
Article 43 comes under the Directive principles based on Gandhian ideology.

92. Consider the following statements:


1. The fundamental Duties in the Indian Constitution are inspired by the Constitution of the United
States of America.
2. The Constitution of India is the only democratic Constitution in the world which contains a list of
duties of citizens.

Which of the statements given above is/are correct?


A. 1 only
B. 2 only 7
C. Both 1 and 2 277
6 3
D. Neither 1 nor 2
560
- 9
Your Answer : D m
Correct Answer : D il.co
m a
4 ;g
Answer Justification : 6
5 &#
a9 in the Indian Constitution are inspired by the
Option (d) is correct: The Fundamental Duties
v
Constitution of erstwhile USSR. ta
i vas
sr
ti of major democratic countries like USA, Canada, France,
Notably, none of the Constitutions
u
r specifically contain a list of duties of citizens.
Germany, Australia and sohon
a -s
tav is, perhaps, the only democratic Constitution in world which contains a
Japanese Constitution
s
list of dutiesvaof citizens.
S ri
i
The
r utsocialist countries, on the contrary, gave equal importance to the fundamental rights and
Shduties of their citizens.

93. In the Indian Political context, the Swaran Singh Committee recommended the-

A. Inclusion of a separate chapter on fundamental duties in the Constitution.


B. Abolition of legislative councils in states.
C. Abolition of ratification by State Legislatures to amend the Constitution.
D. Limited power of Parliament to amend the constitution.

Your Answer : A
Correct Answer : A

Answer Justification :

instacourses.insightsonindia.com 70
© Insights Active Learning | All rights reserved - 18107. You may not reproduce, distribute or exploit the contents in any
form without written permission by copyright owner. Copyright infringers may face civil and criminal liability
Total Marks : 200.00
Prelims Test 23 - GS Test 16
( Insta Prelims Test Series 2024 7.0 ) Mark Scored : 86.90

Option (a) is correct: In 1976, the Congress Party set up the Sardar Swaran Singh Committee
to make recommendations about fundamental duties, the need and necessity of which was felt
during the operation of the internal emergency (1975–1977).

The committee recommended the inclusion of a separate chapter on fundamental duties


in the Constitution.
It stressed that the citizens should become conscious that in addition to the enjoyment of
rights, they also have certain duties to perform as well.

94. Consider the following statements regarding the Amendment of the Indian
Constitution:
1. The Constitution does not prescribe the time frame within which state legislatures should ratify
an amendment submitted to them.
2. The power to initiate an amendment lies with the Parliament and state legislatures cannot 7
7
initiate any bill for amending the Constitution under any circumstance. 27
63
Which of the statements given above is/are correct? 560
- 9
A. 1 only m
B. 2 only
il.co
C. Both 1 and 2 m a
D. Neither 1 nor 2 4 ;g
6
5 &#
Your Answer : C
va9
Correct Answer : A ta
i vas
i sr
ut
Answer Justification :
r
sh
Option (a) is correct:- The Constitution does not prescribe the time frame within which the
state legislaturest a va ratify or reject an amendment submitted to them. Also, it is silent on
should
v as the states can withdraw their approval after according the same.
the issue whether
i
i Sr
t
h ru power to initiate an amendment to the Constitution lies with the Parliament. Hence, unlike
The
S in USA, the state legislatures cannot initiate any bill or proposal for amending the
Constitution except in one case, that is, passing a resolution requesting the
Parliament for the creation or abolition of legislative councils in the states.

95. Consider the following statements:


1. "'The concept of judicial review’ and ‘the Federal character of the State’ are crucial elements of
the Basic Structure of the Constitution.
2. The Fundamental Rights cannot be amended under Article 368 by the Indian Parliament
because it alters the ‘basic structure’ of the Constitution.

Which of the statements given above is/are correct?


A. 1 only
B. 2 only

instacourses.insightsonindia.com 71
© Insights Active Learning | All rights reserved - 18107. You may not reproduce, distribute or exploit the contents in any
form without written permission by copyright owner. Copyright infringers may face civil and criminal liability
Total Marks : 200.00
Prelims Test 23 - GS Test 16
( Insta Prelims Test Series 2024 7.0 ) Mark Scored : 86.90

C. Both 1 and 2
D. Neither 1 nor 2

Your Answer : A
Correct Answer : A

Answer Justification :

Option (a) is correct: The present position is that the Parliament under Article 368 can amend
any part of the Constitution including the Fundamental Rights but without affecting the ‘basic
structure’ of the Constitution.

However, the Supreme Court is yet to define or clarify as to what constitutes the ‘basic
structure’ of the Constitution.
7
From the various judgements, the following have emerged as ‘basic features’ of the
277
Constitution or ingredients of the ‘basic structure’ of the constitution: 63
560
Supremacy of the Constitution - 9
m
i l .co
Sovereign, democratic and republican nature of the Indian polity
Secular character of the Constitution
m a
;g
Separation of powers between the legislature, the executive
4
and the judiciary
Federal character of the Constitution
& #6
Unity and integrity of the nation 5
Welfare state (socio-economic justice) va9
ta
Judicial review as iv
sr
ti
ru
96. Under Second Schedulesh
a - theofemoluments,
the constitution, for how many of the following have the
provisions relating
t a v to allowances and privileges has been
mentioned? s
a
rivof the High Courts
1. The Judges
S
i ministers
2. The tUnion
r u
3. hThe members of Parliament
S
4. Deputy Chairman of the Legislative Council in the states

Select the correct answer using the code given below:


A. Only one
B. Only two
C. Only three
D. All four

Your Answer : C
Correct Answer : B

Answer Justification :

Second Schedule

instacourses.insightsonindia.com 72
© Insights Active Learning | All rights reserved - 18107. You may not reproduce, distribute or exploit the contents in any
form without written permission by copyright owner. Copyright infringers may face civil and criminal liability
Total Marks : 200.00
Prelims Test 23 - GS Test 16
( Insta Prelims Test Series 2024 7.0 ) Mark Scored : 86.90

Provisions relating to the emoluments, allowances, privileges and so on of:

1. The President of India

2. The Governors of States

3. The Speaker and the Deputy Speaker of the Lok Sabha

4. The Chairman and the Deputy Chairman of the Rajya Sabha

5. The Speaker and the Deputy Speaker of the Legislative Assembly in the states

6. The Chairman and the Deputy Chairman of the Legislative Council in the states

7. The Judges of the Supreme Court


7
8. The Judges of the High Courts 277
6 3
9. The Comptroller and Auditor-General of India 560
- 9
m
il.co
m a
97. The idea of “the first past the post” system in Indian g
4 ; election was borrowed from the
Constitution of 6 #
&
a95
tav
A. France
B. Australia
i vas
C. Britain
tisr
u
hr
D. Canada
- s
Your Answer : C ava
t
as : C
Correct Answer
Sriv
i
ut Justification :
Answer
r
Sh
This diagram contains all major borrowings:

instacourses.insightsonindia.com 73
© Insights Active Learning | All rights reserved - 18107. You may not reproduce, distribute or exploit the contents in any
form without written permission by copyright owner. Copyright infringers may face civil and criminal liability
Total Marks : 200.00
Prelims Test 23 - GS Test 16
( Insta Prelims Test Series 2024 7.0 ) Mark Scored : 86.90

7
277
6 3
560
- 9
m
il.co
m a
4 ;g
6
5 &#
va9
ta
i vas
tisr
u
shr
a -
stav
a
Sriv
i
r ut
Sh

98. Consider the following statements


1. The term ‘Federation’ has been clearly defined in the Article 1 of the Indian Constitution.
2. A state has the right to secede from the federation if it passes a bill in the state assembly with
100 % of total members voting in favour of it.

Which of the statements given above is/are correct?


A. 1 only
B. 2 only
C. Both 1 and 2
D. Neither 1 nor 2

Your Answer : D
instacourses.insightsonindia.com 74
© Insights Active Learning | All rights reserved - 18107. You may not reproduce, distribute or exploit the contents in any
form without written permission by copyright owner. Copyright infringers may face civil and criminal liability
Total Marks : 200.00
Prelims Test 23 - GS Test 16
( Insta Prelims Test Series 2024 7.0 ) Mark Scored : 86.90

Correct Answer : D

Answer Justification :

The term ‘Federation’ has nowhere been used in the Constitution. Hence Statement 1 is
incorrect.

Article 1, on the other, describes India as a ‘Union of States’ which implies two things: one,
Indian Federation is not the result of an agreement by the states; and two, no state has the
right to secede from the federation. Hence Statement 2 is incorrect.

That’s why, Indian Constitution has been variously described as ‘federal in form but unitary in
spirit’, ‘quasi-federal’ by K C Wheare.

7 77
2
99. Convention no. 185 of the International Labour Organisation (ILO), often seen
0 63 in the
56
news, deals with?
- 9
m
.co
A. Child labour
B. Equal treatment of labour ai l
g m
4;
C. Seafarer Identity Document
D. Gender equality in wages 6
5 &#
Your Answer : A va9
st a
Correct Answer : C
va
i sri
t
Answer Justification :
h ru
a -s
ILO Convention No.v185, also known as the Seafarers' Identity Documents Convention (Revised),
sta international treaty for all Members that ratify it. It is the first
2003, is a binding
a
riv binding instrument providing for a global mandatory identification system for a
international
S
i
ut of workers. The convention requires an internationally interoperable biometric to be
category
r
Shused for verification of seafarer identities

100. Which of the following curve depicts the relationship between distribution of income
and inequality in an economy?

A. Philips Curve
B. Laffer Curve
C. Lorenz Curve
D. Kuznet Curve

Your Answer : C
Correct Answer : C

instacourses.insightsonindia.com 75
© Insights Active Learning | All rights reserved - 18107. You may not reproduce, distribute or exploit the contents in any
form without written permission by copyright owner. Copyright infringers may face civil and criminal liability
Total Marks : 200.00
Prelims Test 23 - GS Test 16
( Insta Prelims Test Series 2024 7.0 ) Mark Scored : 86.90

Answer Justification :

The Lorenz curve is a graphical representation of income inequality or wealth inequality


developed by American economist Max Lorenz in 1905. The graph plots percentiles of the
population on the horizontal axis according to income or wealth. Hence, option (c) is correct.

7
277
6 3
560
- 9
m
il.co
m a
4 ;g
6
5 &#
va9
ta
i vas
tisr
u
shr
a -
stav
a
Sriv
i
r ut
Sh

instacourses.insightsonindia.com 76
© Insights Active Learning | All rights reserved - 18107. You may not reproduce, distribute or exploit the contents in any
form without written permission by copyright owner. Copyright infringers may face civil and criminal liability

You might also like